You are on page 1of 100

SUGGESTED ANSWERS TO QUESTIONS SET AT THE

COMMON FOR

INTEGRATED PROFESSIONAL COMPETENCE EXAMINATION GROUP I & ACCOUNTING TECHNICIAN EXAMINATION


MAY, 2011

BOARD OF STUDIES THE INSTITUTE OF CHARTERED ACCOUNTANTS OF INDIA


(Set up by an Act of Parliament)

The suggested Answers published in this volume do not constitute the basis for evaluation of the students answers in the examination. The answers are prepared by the Faculty of the Board of Studies with a view to assist the students in their education. While due care is taken in preparation of the answers, if any errors or omissions are noticed, the same may be brought to the attention of the Director of Studies. The Council of the Institute is not in anyway responsible for the correctness or otherwise of the answers published herein.

THE INSTITUTE OF CHARTERED ACCOUNTANTS OF INDIA

All rights reserved. No part of this publication may be reproduced, stored in a retrieval system, or transmitted, in any form, or by any means, electronic, mechanical, photocopying, recording, or otherwise, without prior permission, in writing, form the publisher.

Website Committee/ Department E-mail Price ISBN No. Published by

: : : : : :

www.icai.org Board of Studies bosnoida@icai.org ` 40/978-81-8441-460-8 The Publication Department on behalf of The Institute of Chartered Accountants of India, ICAI Bhawan, Post Box No. 7100, Indraprastha Marg, New Delhi- 110 002, India Typeset and designed at Board of Studies.

Printed by

Repro India Limited, Plot No. 50/2, T.T.C. MIDC Industrial Area, Mahape, Navi Mumbai 400 710. August/2011/58000 copies

Contents
Page Nos. Paper 1. Paper 2. Paper 3. Paper 4 Accounting ........................................................................................... 1 24 Business Laws, Ethics and Communication .........................................25 39 Cost Accounting and Financial Management .......................................40 63 Taxation ..............................................................................................64 89

Summary of Examiners comments on the performance of the candidates

PAPER 1 : ACCOUNTING Question No. 1 is compulsory Answer any five questions from the remaining six questions. Wherever necessary suitable assumptions should be made by the candidates. Working Notes should form part of the answer. Question 1 Answer the following question: (a) The abstract of the Balance Sheet of the AXE Ltd. as at 31st March 2011, are as follows: Liabilities Equity share capital (` 100 each) 12% Preference share capital (` 100 each)

`
15,00,000 8,00,000

13% Debentures 3,00,000 st On 31 March, 2011, BXE Ltd. agreed to take over AXE Ltd. on the following terms: (1) For each preference share in AXE Ltd., ` 10 in cash and one 9% preference share of ` 100 in BXE Ltd. (2) For each equity share AXE Ltd. ` 20 in cash and one equity share in BXE Ltd. of ` 100 each. It was decided that the share in BXE Ltd. will be issued at market price ` 140 per share. (3) Liquidation expenses of AXE Ltd. are to be reimbursed by BXE Ltd. to the extent of ` 10,000. Actual expenses amounted to ` 12,500. You are required to compute the amount of purchase consideration. (b) On 30th March, 2011 fire occurred in the premises of M/s Suraj Brothers. The concern had taken an insurance policy of ` 60,000 which was subject to the average clause. From the books of accounts, the following particulars are available relating to the period 1st January to 30th March 2011. (1) Stock as per Balance Sheet at 31st December, 2010, ` 95,600. (2) Purchases (including purchase of machinery costing ` 30,000) ` 1,70,000 (3) Wages (including wages ` 3,000 for installation of machinery) ` 50,000. (4) Sales (including goods sold on approval basis amounting to ` 49,500) ` 2,75,000. No approval has been received in respect of 2/3rd of the goods sold on approval. (5) The average rate of gross profit is 20% of sales. (6) The value of the salvaged goods was ` 12,300.

INTEGRATED PROFESSIONAL COMPETENCE EXAMINATION : MAY, 2011

You are required to compute the amount of the claim to be lodged to the insurance company. (c) Shiv and Mohan are partners in a firm sharing profits and losses equally. On 31st March, 2011, the balances of their capital accounts were ` 3,00,000 and ` 2,00,000 respectively. The average profits of the firm are ` 1,36,000 and the rate of normal profit is 20%. On 1st April, 2011 they agreed to admit Hari as a partner for one fourth share. Hari will bring ` 1,00,000 as capital. You are required to compute the value of the goodwill of the firm on admission of Hari, if goodwill is to be calculated on the basis of: (1) 5 years purchase of super profit (2) Capitalization method (3) 3 years purchase of average profit. (d) On 1st April, 2010, Rajat has 50,000 equity shares of P Ltd. at a book value of ` 15 per share (face value ` 10 each). He provides you the further information: (1) On 20th June, 2010, he purchased another 10,000 shares of P Ltd. at ` 16 per share. (2) On 1st August, 2010, P Ltd. issued one equity bonus share for every six shares held by the shareholders. (3) On 31st October, 2010, the directors of P Ltd. announced a right issue which entitle the holders to subscribe three shares for every seven shares at ` 15 per share. Shareholders can transfer their rights in full or in part. Rajat sold 1/3rd of entitlement to Umang for a consideration of ` 2 per share and subscribe the rest on 5th November, 2010. You are required to prepare Investment A/c in the books of Rajat for the year ending 31st March, 2011. (4 5 = 20 Marks) Answer (a) Calculation of purchase consideration ` I Payment made to shareholders of 8,000 preference shares of AXE Ltd. : Cash @ ` 10 per share (8,000 preference shares x ` 10)

80,000

8,00,000 = 8,000 preference shares 100


2

PAPER 1 : ACCOUNTING

9% Preference shares in BXE Ltd. @ ` 100 each II Payment made to Equity shareholders of 15,000 equity shares of AXE Ltd. : Cash @ ` 20 per share (15,000 shares x ` 20) Equity shares in BXE Ltd. issued at market price ` 140 each (15,000 shares x ` 140) Total purchase consideration

8,00,000

8,80,000

3,00,000 21,00,000 24,00,000 32,80,000

Note: Re-imbursement of liquidation expenses of AXE Ltd. to the extent of ` 10,000, will not be included in the calculation of purchase consideration. (b) Computation of claim for loss of stock ` Stock on the date of fire i.e. on 30th March, 2011 (W.N.1) Less: Value of salvaged stock Loss of stock Amount of claim =
Insured value Total cost of stock on the date of fire

62,600 (12,300) 50,300 48,211 (approx.)

x Loss of stock

60,000 50,300 = 62,600


A claim of ` 48,211 (approx.) should be lodged by M/s Suraj Brothers to the insurance company. Working Notes: 1. Calculation of closing stock as on 30th March, 2011 Memorandum Trading Account for (from 1st January, 2011 to 30th March, 2011) Particulars To To To Opening stock Purchases (1,70,00030,000) Wages (50,000 Amount Particulars (`) 95,600 By By 1,40,000 By Sales (W.N.3) Goods with customers (for approval) (W.N.2) Closing stock (Bal. fig.) Amount (`) 2,42,000 26,400 62,600

15,00,000 = 15,000 equity shares 100


3

INTEGRATED PROFESSIONAL COMPETENCE EXAMINATION : MAY, 2011

3,000) To Gross profit (20% on sales)

47,000 48,400 3,31,000

2.

3,31,000 Calculation of goods with customers

Since no approval for sale has been received for the goods of ` 33,000 (i.e. 2/3 of ` 49,500) hence, these should be valued at cost i.e. ` 33,000 20% of ` 33,000 = ` 26,400. 3. Calculation of actual sales Total sales Sale of goods on approval = ` 2,75,000 ` 33,000 = ` 2,42,000. (c) Valuation of goodwill (1) 5 years purchase of super profit ` Average profit Less : Normal profit @ 20% of (` 3,00,000+ ` 2,00,000) Super profit Value of goodwill = 5 Super profit = 5 ` 36,000 = ` 1,80,000 Value of goodwill of the firm will be ` 1,80,000. (2) Capitalisation method Normal value of business = =
Average profit Normal rate of profit

1,36,000 (1,00,000) 36,000

1,36,000 = ` 6,80,000 20%

`
Normal value of business Less: Actual capital employed Shiv Mohan Value of goodwill of the firm will be (3) 3 years purchase of average profits Goodwill = 3 Average profit
4

6,80,000 3,00,000 2,00,000 (5,00,000) 1,80,000

PAPER 1 : ACCOUNTING

= 3 ` 1,36,000 = ` 4,08,000 Value of goodwill of the firm will be ` 4,08,000. (d) In the books of Rajat Investment Account (Equity shares in P Ltd. )
Date 1.4.10 20.6.10 1.8.10 5.11.10 Particulars To Balance b/d To Bank A/c To Bonus issue (W.N.1) To Bank A/c (right shares) (W.N.4) No. of shares 50,000 10,000 10,000 Amount Date (`) 7,50,000 5.11.10 1,60,000 - 31.3.11 Particulars By Bank A/c (sale of rights) (W.N.3) By Balance c/d (Bal. fig.) No. of shares Amount (`)

20,000 90,000 11,90,000

20,000 3,00,000 90,000 12,10,000

90,000 12,10,000

Working Notes: (1) Bonus shares = (2) Right shares =

50,000 + 10,000 = 10,000 shares 6 50,000 + 10,000 + 10,000 3 = 30,000 shares 7

1 (3) Sale of rights = 30,000 shares ` 2= ` 20,000 3 2 (4) Rights subscribed = 30,000 shares ` 15 = ` 3,00,000 3
Question 2 Amit and Sumit are partners sharing profits and losses in the ratio of 3:2. Their Balance Sheet as on 31st March 2011 is given below: Liabilities Capital Accounts: Amit Sumit Loan from Puneet Amount Assets Amount

`
Land & building 1,76,000 Investments (Market value ` 55,000) 2,54,000 Debtors 3,00,000 3,00,000 Less: Provision for doubtful debts 10,000
5

`
3,20,000 50,000 2,90,000

INTEGRATED PROFESSIONAL COMPETENCE EXAMINATION : MAY, 2011

General Reserve Employers provident fund Creditors

30,000 Stock 1,10,000 10,000 Cash at bank 50,000 50,000 8,20,000 8,20,000 st They decided to admit Puneet as a new partner from 1 April, 2011 on the following terms: (1) Amit will give 1/3rd of his share and Sumit will give 1/4th of his share to Puneet. (2) Puneets loan account will be converted into his capital. (3) The Goodwill of the firm is valued at ` 3,00,000. Puneet will bring his share of goodwill in cash and the same was immediately withdrawn by the partners. (4) Land and building was found undervalued by ` 1,00,000. (5) Stock was found overvalued by ` 60,000. (6) Provision for doubtful debts will be made equal to 5% of debtors. (7) Investments are to be valued at their market price. It was decided that the total capital of the firm after admission of new partner would be

` 10,00,000. Capital accounts of partners will be readjusted on the basis of their profit sharing
ratio and excess or deficiency will be adjusted in cash. You are required to prepare: (a) Revaluation A/c (b) Partners capital A/cs (c) Balance Sheet of the firm after admission of a new partner Answer Revaluation A/c Particulars (16 Marks)

` Particulars
60,000 By Land & building 5,000 By Investments 24,000 16,000 1,05,000

`
1,00,000 5,000

To Stock To Provision for doubtful debts To Profit transferred to Amits capital A/c Sumits capital A/c

1,05,000

PAPER 1 : ACCOUNTING

Partners Capital Accounts


Particulars To Amits capital A/c To Puneets capital A/c To Bank A/c To Balance c/d Amit Sumit Puneet Particulars Amit Sumit Puneet

`
60,000 4,00,000

`
30,000 3,00,000

`
By Balance b/d 60,000 By Puneets' Loan A/c 30,000 By Puneets capital A/c - By Bank A/c (W.N.2) 3,00,000 By Revaluation A/c By General reserve By Bank

1,76,000 2,54,000 - 3,00,000 60,000 24,000 30,000 16,000 90,000 -

18,000 1,82,000

12,000 18,000

4,60,000

3,30,000

3,90,000

4,60,000 3,30,000 3,90,000

Balance Sheet as on 1st April, 2011 (After admission of a new partner - Puneet) Liabilities Amount Assets ` Amount `

Capital accounts Amit Sumit Puneet Creditors Employers provident fund


Working Notes:

Land and building (3,20,000 + 1,00,000) 4,00,000 Investments 3,00,000 Debtors 50,000 Stock (1,10,000 60,000) 10,000 10,60,000 Cash at bank (W.N. 3) 3,00,000 3,00,000 Less: Provision for doubtful debts (15,000)

4,20,000 55,000 2,85,000 50,000 2,50,000 10,60,000

(1) Calculation of incoming partners share, new profit sharing ratio and sacrificing ratio Amit Sumit

Old profit sharing ratio

3/5

2/5

It is assumed that Employers Provident Fund represents employers contribution to provident fund which is yet to be deposited. Hence, the same represents a current liability. 7

INTEGRATED PROFESSIONAL COMPETENCE EXAMINATION : MAY, 2011

Surrendered by old partners Remaining share Puneets total share in profits = 1/5 + 1/10 = 3/10

3/5 x 1/3 = 1/5 3/5 1/5 = 2/5

2/5 x 1/4 = 1/10 2/5 1/10 = 3/10

New profit sharing ratio of Amit : Sumit : Puneet =2/5 : 3/10 : 3/10 = 4:3:3 Sacrificing ratio of Amit : Sumit is 1/5 : 1/10 : or 2:1
(2) Calculation of share of goodwill by old partners

Goodwill of the firm was ` 3,00,000 Share of Puneet in goodwill = ` 3,00,000

3 = ` 90,000 10

Goodwill will be distributed among the old partners in their sacrificing ratio of 2:1 i.e. ` 60,000 by Amit and ` 30,000 by Sumit.
(3) Calculation of closing balance of bank account after admission Bank A/c Particulars Amount Particulars (` ) Amount (` )

To Balance b/d To Puneets capital A/c To Sumits capital A/c To Amits capital A/c
Question 3

50,000 By Amits capital A/c 90,000 By Sumits capital A/c 18,000 By Balance c/d 1,82,000 3,40,000

60,000 30,000 2,50,000 3,40,000

The Balance Sheet of Mars Limited as on 31st March, 2011 was as follow: Liabilities Share Capital: 1,00,000 Equity shares of ` 10 each fully paid up Reserve and surplus Capital reserve Contingency reserve Profit and loss A/c

` Assets
Fixed Assets: Land and building 10,00,000 Current Assets Stock 42,000 Sundry debtors 2,70,000 Less : Provision for 2,52,000 doubtful debts
8

`
7,64,000 7,75,000 1,60,000 1,52,000 8,000

PAPER 1 : ACCOUNTING

Current Liabilities & Provisions Bills payable Sundry creditors Provisions for income tax 2,26,000 2,20,000 20,50,000

Bill receivable 40,000 Cash at bank

30,000 3,29,000

20,50,000

On 1st April, 2011, Jupiter Limited agreed to absorb Mars Limited on the following terms and conditions: (1) Jupiter Limited will take over the assets at the following values:

`
Land and building Stock 10,80,000 7,70,000

Bills receivable 30,000 (2) Purchase consideration will be settled by Jupiter Ltd. as under: 4,100 fully paid 10% preference shares of ` 100 will be issued and the balance will be settled by issuing equity shares of `10 each at ` 8 paid up. (3) Liquidation expenses are to be reimbursed by Jupiter Ltd. to the extent of ` 5,000. (4) Sundry debtors realized ` 1,50,000. Bills payable were settled for ` 38,000. Income tax authorities fixed the taxation liability at ` 2,22,000 and the same was paid. (5) Creditors were finally settled with cash remaining after meeting liquidation expenses amounting to ` 8,000 You are required to: (i) (ii) Calculate the number of equity shares and preference shares to be allotted by Jupiter Limited in discharge of purchase consideration Prepare the Realisation account, Bank account, Equity shareholders account and Jupiter Limiteds account in the books of Mars Ltd. (16 Marks) Calculation of number of shares to be allotted Particulars Amount (`)

Answer (i)

Land and building Stock Bills receivable Total

10,80,000 7,70,000 30,000 18,80,000

INTEGRATED PROFESSIONAL COMPETENCE EXAMINATION : MAY, 2011

Amount discharged by issue of preference shares Number of preference shares to be issued (4,10,000/100) Amount discharged by issue of equity shares (` 18,80,000 4,10,000) Number of equity shares to be issued (` 14,70,000 / 8)
(ii) Particulars Ledger Accounts in the books of Mars Limited Realization Account

4,10,000 4,100 shares 14,70,000 1,83,750 Shares

To Land and building To Stock To Sundry debtors To Bills receivable To Bank A/c liquidation expenses To Bank A/c- bills payable To Bank A/c income tax To Bank A/c sundry creditors To Profit transferred to equity shareholders A/c

` 7,64,000 7,75,000 1,60,000 30,000 3,000

Particulars

By Provision for doubtful debts By Bills payable By Sundry creditors By Provision for taxation By Jupiter Ltd. (purchase consideration)

` 8,000 40,000 2,26,000 2,20,000


18,80,000 1,50,000

38,000 By Bank A/c- sundry debtors 2,22,000 2,16,000 3,16,000 25,24,000


Bank Account

25,24,000

Particulars

` Particulars

To Balance b/d To Realisation A/c (payment received from debtors) To Jupiter Ltd. (liquidation expenses)

3,29,000 By Realisation A/c (liquidation expenses) 1,50,000 By Jupiter Ltd. 5,000 By Bills payable By Income tax By Sundry creditors (Bal.fig.) 4,84,000

3,000 5,000 38,000 2,22,000 2,16,000 4,84,000

Equity Shareholders Account Particulars

To 10% Preference shares

` Particulars 4,10,000 By Equity share capital A/c


10

` 10,00,000

PAPER 1 : ACCOUNTING

in Jupiter Limited To Equity shares in Jupiter Limited

By Capital reserve 14,70,000 By Contingency reserve By Profit and loss A/c By Realisation A/c (profit) 18,80,000
Jupiter Limited Account

42,000 2,70,000 2,52,000 3,16,000 18,80,000

Particulars

To Realisation A/c

` Particulars 18,80,000 To 10% Preference shares in Jupiter Limited To Equity shares in Jupiter Limited 18,80,000

` 4,10,000

14,70,000 18,80,000

Question 4 The following are the summarized Balance Sheets of Lotus Ltd. as on 31st March 2010 and 2011: Liabilities Equity share capital (` 10 each) Capital reserve Profit and loss A/c Long term loan from the bank Sundry creditors Provision for taxation Assets Land and building Machinery Investment Stock Sundry debtors Cash in hand Cash at bank 31-3-2010 31-3-2011

`
10,00,000 4,00,000 5,00,000 5,00,000 50,000 24,50,000

`
12,50,000 10,000 4,80,000 4,00,000 4,00,000 60,000 26,00,000

`
4,00,000 7,50,000 1,00,000 3,00,000 4,00,000 2,00,000 3,00,000 24,50,000

`
3,80,000 9,20,000 50,000 2,80,000 4,20,000 1,40,000 4,10,000 26,00,000

11

INTEGRATED PROFESSIONAL COMPETENCE EXAMINATION : MAY, 2011

Additional information: (1) Depreciation written off on land and building ` 20,000. (2) The company sold some investment at a profit of ` 10,000, which was credited to Capital Reserve. (3) Income-tax provided during the year ` 55,000. (4) During the year, the company purchased a machinery for ` 2,25,000. They paid ` 1,25,000 in cash and issued 10,000 equity shares of ` 10 each at par. You are required to prepare a cash flow statement for the year ended 31st March 2011 as per AS 3 by using indirect method. (16 Marks) Answer In the books of Lotus Ltd. Cash Flow Statement for the year ending 31st March, 2011

`
I Cash flow from Operating Activities

Net Profit before tax for the year (W.N.1)


Add: Depreciation on machinery (W.N.2)

1,35,000 55,000 20,000 2,10,000 20,000 (20,000) (1,00,000) 1,10,000 (45,000) 65,000 (1,25,000) 60,000 (65,000) 1,50,000 (1,00,000) 50,000 50,000
12

Depreciation on land & building Operating profit before change in working capital
Add: Decrease in stock Less: Increase in sundry debtors Less: Decrease in sundry creditors

Cash generated from Operations


Less: Income tax paid (W.N.3)

Net cash generated from operating activities


II Cash flow from Investing activities

Purchase of machinery (2,25,000 1,00,000) Sale of investment (W.N. 4) Net cash used in investing activities
III Cash flow from financing activities

Issue of equity shares (2,50,000-1,00,000) Repayment of long term loan Net cash generated from financing activities Net increase in cash and cash equivalents

PAPER 1 : ACCOUNTING

Cash and cash equivalents at the beginning of the year (2,00,000 + 3,00,000) Cash and cash equivalents at the end of the year (1,40,000+4,10,000)
Working Notes: 1. Calculation of Net Profit before tax `

5,00,000 5,50,000

Increase in Profit & Loss (Cr.) balance


Add: Provision for taxation made during the year 2.

80,000 55,000 1,35,000

Calculation of Depreciation charged during the year on Machinery account Particulars To Balance b/d To Bank To Equity share capital Amount (`) Particulars 7,50,000 By Depreciation (Bal.fig.) 1,25,000 By Balance c/d 1,00,000 9,75,000 Amount (`) 55,000 9,20,000

9,75,000

3.

Calculation of tax paid during the year Provision for Taxation A/c Particulars Amount (`) Particulars Amount (`)

To Cash (Bal.fig.) To Balance c/d


4.

45,000 By Balance b/d 60,000 By Profit and Loss A/c 1,05,000

50,000 55,000 1,05,000

Calculation of sales value of investment sold Investment A/c Particulars Amount (`) Particulars Amount (`)

To Balance b/d To Capital reserve (Profit on sale of investments)

1,00,000 By Bank A/c (Bal.fig.) 10,000 1,10,000 By Balance c/d

60,000 50,000 1,10,000

13

INTEGRATED PROFESSIONAL COMPETENCE EXAMINATION : MAY, 2011

Question 5 The following is the Receipt and Payment Account of Park View Club in respect of the year ended 31st March, 2011. Receipt To Balance b/d To Subscriptions 2009-10 2010-11 2011-12

Amount (`) 1,02,500 4,500

Payments By Salaries By Stationery By Rent By Telephone expenses

Amount ( `) 2,08,000 40,000 60,000 10,000 1,25,000 92,500 45,000 5,80,500

2,11,000 7,500 2,23,000 1,55,000 1,00,000 5,80,500

By Investment By Sundry expenses By Balance c/d

To Profit on sports meet To Income from investments Additional information:

(1) There are 450 members each paying an annual subscription of ` 500. On 1st April, 2010 outstanding subscription was ` 5,000. (2) There was an outstanding telephone bill for ` 3,500 on 31st March, 2011. (3) Outstanding sundry expenses as on 31st March, 2010 totalled ` 7,000. (4) Stock of stationery: On 31st March, 2010 On 31st March, 2011

` 5,000 ` 9,000

(5) On 31st March, 2010 building stood in the books at ` 10,00,000 and it was subject to depreciation @ 5% per annum. (6) Investment on 31st March, 2010 stood at ` 20,00,000. (7) On 31st march, 2011, income accrued on the investments purchased during the year amounted to ` 3,750. Prepare an Income and Expenditure Account for the year ended 31st March, 2011 and the Balance Sheet as at that date. (16 Marks)

In the questions paper, the years have been wrongly printed as 2008-09 (instead of 2009-10), 2009-10 (instead of 2010-11) and 2010-11 (instead of 2011-12). However, in the question given above, these corrections have been incorporated.

14

PAPER 1 : ACCOUNTING

Answer Park View Club Income and Expenditure Account for the year ending on 31st March 2011
Expenditure To Salaries To Stationery consumed (W.N.3) To Rent To Telephone expenses 10,000 Add: Outstanding on 31.3.11 3,500 To Sundry expenses To Depreciation of building To Surplus (excess of income over expenditure) 92,500 85,500 50,000 30,750 4,83,750 4,83,750 Less: Outstanding on 31.3.10 (7,000) Amount (`) 2,08,000 36,000 60,000 13,500 Income By Subscriptions (W.N. 2) By Profit on sports meet By Income on investments 1,00,000 Add: Income accrued 3,750 1,03,750 Amount (`) 2,25,000 1,55,000

Balance Sheet as at 31st March 2011


Liabilities Capital fund (W.N.1) 31,05,500 Add: Surplus 30,750 Subscriptions received in advance Outstanding telephone bills 31,36,250 7,500 3,500 Amount (`) Assets Outstanding subscriptions Investment (20,00,000+1,25,000) 21,25,000 Add: Interest accrued on investments 3,750 Building 10,00,000 Less: Depreciation Stock of stationery Cash balance 31,47,250 (50,000) Amount (`) 14,500

21,28,750 9,50,000 9,000 45,000 31,47,250

Working Notes: (1) Liabilities Balance Sheet as at 31st March 2010 Amount (`) Assets Amount (`)

Outstanding sundry expenses

7,000 Building
15

10,00,000

INTEGRATED PROFESSIONAL COMPETENCE EXAMINATION : MAY, 2011

Capital fund (Bal.fig.)

31,05,500 Investments Stock of stationery Cash balance Outstanding subscriptions 31,12,500

20,00,000 5,000 1,02,500 5,000 31,12,500

(2) Calculation of subscriptions accrued during the year Subscription A/c Particulars Amount Particulars (`) Amount (` )

To Outstanding Subscriptions (as on 1.4.10) To Income & Expenditure A/c To Subscriptions received in advance for 2011-12

5,000 By Cash A/c By Outstanding subscriptions 2,25,000 7,500 (as on 31.3.11) (Bal.fig.)

2,23,000 14,500

2,37,500 (3) Calculation of stationery consumed during the year Stock of stationery as on 31 March, 2010
Add: Purchased during the year 2010-11 Less: Stock of stationery as on 31st March, 2011

2,37,500

Stationery consumed
Question 6

` 5,000 40,000 45,000 (9,000) 36,000

Mr A runs a business of readymade garments. He closes the books of accounts on 31st March, 2010. The Balance Sheet as on 31st March, 2010 was as follows: Liabilities

As capital a/c Creditors

` Assets 4,04,000 Furniture 82,000 Stock Debtors Cash in hand Cash at bank 4,86,000
16

` 40,000 2,80,000 1,00,000 28,000 38,000 4,86,000

PAPER 1 : ACCOUNTING

You are furnished with the following information: (1) His sales, for the year ended 31st March, 2011 were 20% higher than the sales of previous year, out of which 20% sales was cash sales. Total sales during the year 2009-10 were ` 5,00,000. (2) Payments for all the purchases were made by cheques only. (3) Goods were sold for cash and credit both. Credit customers pay be cheques only. (4) Deprecition on furniture is to be charged 10% p.a. (5) Mr A sent to the bank the collection of the month at the last date of the each month after paying salary of ` 2,000 to the clerk, office expenses `1,200 and personal expenses ` 500. Analysis of bank pass book for the year ending 31st March 2011 disclosed the following:

`
Payment to creditors Payment of rent up to 31st March, 2011 Cash deposited into the bank during the year The following are the balances on 31st March, 2011: Stock Debtors 3,00,000 16,000 80,000

`
1,60,000 1,20,000

Creditors for goods 1,46,000 st On the evening of 31 March 2011, the cashier absconded with the available cash in the cash book. You are required to prepare Trading and Profit and Loss A/c for the year ended 31st March, 2011 and Balance Sheet as on that date. All the workings should form part of the answer. (16 Marks) Answer Trading and Profit and Loss Account for the year ending 31st March 2011 Particulars

To Opening stock To Purchases (W.N. 1) To Gross profit

` Particulars 2,80,000 By Sales (W.N. 3) 3,64,000 Credit 4,80,000 1,16,000 Cash 1,20,000 By Closing stock 7,60,000
17

6,00,000 1,60,000 7,60,000

INTEGRATED PROFESSIONAL COMPETENCE EXAMINATION : MAY, 2011

To Salary To Rent To Office expenses To Loss of cash (W.N. 6) To Depreciation on furniture To Net Profit

24,000 By Gross profit 16,000 14,400 23,600 4,000 34,000 1,16,000


Balance Sheet as on 31st March, 2011

1,16,000

1,16,000

Liabilities

As Capital
Add: Net Profit Less: Drawings

4,04,000 34,000 (6,000)

Creditors
Working Notes: (1) Calculation of purchases

` Assets Furniture Less: Depreciation 4,32,000 Stock Debtors 1,46,000 Cash at bank 5,78,000

`
40,000 (4,000) 36,000 1,60,000 1,20,000 2,62,000 5,78,000

Creditors Account Particulars

` Particulars
3,00,000 By Balance b/d 1,46,000 4,46,000 By Purchases (Bal.fig.)

`
82,000 3,64,000 4,46,000

To Bank A/c To Balance c/d


(2) Calculation of total sales

`
Sales for the year 2009-10
Add: 20% increase

5,00,000 1,00,000 6,00,000

Total sales for the year 2010-11


(3) Calculation of credit sales

`
Total sales 6,00,000

18

PAPER 1 : ACCOUNTING

Less: Cash sales (20% of total sales) (4) Calculation of cash collected from debtors Debtors Account Particulars

(1,20,000) 4,80,000

To Balance b/d To Sales A/c

` Particulars 1,00,000 By Bank A/c (Bal. fig.) 4,80,000 By Balance c/d 5,80,000
Bank Account

` 4,60,000 1,20,000 5,80,000

(5) Calculation of closing balance of cash at bank Particulars

To Balance b/d To Debtors A/c To Cash A/c

` 38,000 4,60,000 80,000 5,78,000

Particulars

By Creditors A/c By Rent A/c By Balance c/d

` 3,00,000 16,000 2,62,000 5,78,000

(6) Calculation of the amount of cash defalcated by the cashier

Cash balance as on

1st

April 2010

Add: Cash sales during the year Less:Salary (` 2,000x12)

` 28,000 1,20,000 1,48,000


24,000 14,400 6,000 80,000 (1,24,400) 23,600

Office expenses (` 1,200 x 12) Drawings of A (` 500x12) Cash deposited into bank during the year Cash balance as on 31st March 2011 (defalcated by the cashier)
Question 7 Answer any four of the following:

(a) A and B are partners in a firm and share profits and losses equally. A has withdrawn the following sum during the half year ending 30th June 2010:

19

INTEGRATED PROFESSIONAL COMPETENCE EXAMINATION : MAY, 2011

Date January 15 February 10 April 5 May 20

Amount

`
5,000 4,000 8,000 10,000

June 18 9,000 Interest on drawings is charged @ 10% per annum. Find out the average due date and calculate the interest on drawings to be charged on 30th June 2010. (b) Best Ltd. deals in five products, P, Q, R, S, and T which are neither similar nor interchangeable. At the time of closing of its accounts for the year ending 31st March 2011, the historical cost and net realizable value of the items of the closing stock are determined as follows: Items P Q R S Historical cost Net realizable value

`
5,70,000 9,80,000 3,16,000 4,25,000

`
4,75,000 10,32,000 2,89,000 4,25,000

T 1,60,000 2,15,000 What will be the value of closing stock for the year ending 31st March, 2011 as per AS 2 Valuation of Inventories? (c) X,Y and Z are partners sharing profits an losses in the ratio of 4:3:2 respectively. On 31st March, 2011 Y retires and X and Z decide to share profits and losses in the ratio of 5:3. Then immediately, W is admitted for 3/10th shares in profits, 2/3rd of which was given by X and rest was taken by W from Z . Goodwill of the firm is valued at ` 2,16,000 W brings required amount of goodwill. Give necessary Journal Entries to adjust goodwill on retirement of Y and admission of W if they do not want to raise goodwill in the books of accounts. (d) In business today, the accounts which were earlier maintained in a manual form, are replaced with computerized accounts. Explain the significance of computerized accounting system in modern time. (e) On 1st October, 2010, the debit balances of debtors account is ` 77,500 in the books of M/s Zee Ltd. Transactions during the 6 months ended on 31st March 2011 were as follows:

20

PAPER 1 : ACCOUNTING

`
Total sales (including cash sales ` 14,000) Payment received from debtors in cash Bills receivable received Discount allowed to customers for prompt payment Goods rejected and returned back by the customer Bad debts recovered (written off in 2009) Interest debited for delay in payment 84,000 38,000 26,000 1,000 2,550 900 1,250

You are required to prepare a Debtors Account for the period ending 31st March in the General of M/s Zee Ltd. (4 4 = 16 Marks) Answer (a) Date Calculation of Average due date (Base Date 15th Jan, 2010) Amount No. of days Product `

`
January 15 February 10 April 5 May 20 June 18 5,000 4,000 8,000 10,000 9,000 36,000 Average due date = Base date + = 15th Jan +
Total product days Total amount

0 26 80 125 154

0 1,04,000 6,40,000 12,50,000 13,86,000 33,80,000

33,80,000 days 36,000

= 15th Jan + 94 days (approx.) = 19th April, 2010 Number of days from 19th April, 2010 to 30th June, 2010 = 72 days Interest on drawings from 19th April to 30th June @10%:

21

INTEGRATED PROFESSIONAL COMPETENCE EXAMINATION : MAY, 2011

= ` 36,000 = ` 710

72 10 365 100

Hence, interest on drawings ` 710 will be charged from A on 30th June, 2010.
(b) As per para 5 of AS 2 Valuation of Inventories, inventories should be valued at the lower of cost and net relizable value. Inventories should be written down to net realizable value on an item-by-item basis. Valuation of inventory (item wise) for the year ending 31st March 2011 Item Historical Cost Net realizable value Valuation of closing stock

P Q R S T

` 5,70,000 9,80,000 3,16,000 4,25,000 1,60,000

` 4,75,000 10,32,000 2,89,000 4,25,000 2,15,000

` 4,75,000 9,80,000 2,89,000 4,25,000 1,60,000 23,29,000

The value of inventory for the year ending 31st March 2011 = ` 23,29,000.
(c) Date Particulars Journal Entries L.F. Dr. (`) Cr.(`)

31.3.11

Xs capital A/c Zs capital A/c To Ys capital A/c (3/9 ` 2,16,000)

Dr. Dr.

39,000 33,000 72,000

(Being Ys share of goodwill adjusted in the capital accounts of gaining partners in their gaining ratio 13:11 Refer Working Note.) Cash A/c Dr. 64,800 64,800 64,800 43,200 21,600 To Ws capital A/c (3/10 ` 2,16,000) (Being the amount of goodwill brought in by W) Ws capital A/c To Xs capital A/c To Zs capital A/c (Being the goodwill credited to sacrificing partners in their sacrificing ratio 2:1)
22

Dr.

PAPER 1 : ACCOUNTING

Working Note: Calculation of gaining ratio of X and Z

Gaining ratio For X Z Gaining ratio

= New ratio Old ratio = 5/8-4/9 = 13/72 = 3/8-2/9 = 11/72 = 13:11

(d) In modern time, computerized accounting systems are used in various areas. The significance of the computerized accounting system is as follows: (1) Increase speed, accuracy and security - In computerized accounting system, the speed with which accounts can be maintained is several fold higher. Besides speed, level of accuracy is also high in computerized accounting system. (2) Reduce errors - In computerized accounting, the possibilities of errors are also very less unless some mistake is made while recording the data. (3) Immediate information - In this system, with an entry of a transaction, corresponding ledger posting is done automatically. Hence, trial balance will also be automatically tallied and the user will get the information immediately. (4) Avoid duplication of work - Computerized accounting systems also remove the duplication of the work. (e)
Date 1.10.10 1.10.10 to 31.3.11

Total Debtors account in the General Ledger of M/s Zee Ltd.


Particulars To Balance c/d To General Ledger Adjustment A/c: Sales (84,000-14,000) Bills receivable (Bill dishonored) Bank (Noting charges) Interest 8,500 250 31.3.11 1,250 1,57,500 23 1,57,500 70,000 Amount Date Particulars to By General Ledger Adjustment A/c: Cash collected 38,000 Amount

`
77,500 1.10.10 31.3.11

Bills Receivable A/c Discount allowed Sales return By Balance c/d

26,000 1,000 2,550 89,950

INTEGRATED PROFESSIONAL COMPETENCE EXAMINATION : MAY, 2011

Working Note:

1. 2.

Bad debts of the year 2008-09 recovered in 2010-11 will not appear in the Total Debtors account. It will be credited to profit & loss account. Bills receivables of ` 5,000 endorsed to the supplier will not be shown in the Total Debtors account because at the time of endorsement Suppliers account will be debited and Bills receivable account will be credited.

24

PAPER 2 : BUSINESS LAWS, ETHICS AND COMMUNICATION Question No. 1 is compulsory. Attempt any five questions from the remaining six questions. Question 1 (a) What is the law relating to determination of compensation, on breach of contract, contained in section 73 of the Indian Contract Act, 1872 ? (5 Marks) (b) (I) State whether the following statements are correct or incorrect.: (i) (ii) (i) A cheque marked Not-Negotiable is not transferable. (3 1= 3 Marks) Which one of the following statements is not true about minors position in the firm: (a) He can not become a partner in the firm. (b) A minor and a major can enter into an agreement of partnership. (c) He can be admitted to the benefits in the firm. (d) He can become a partner on becoming a major. (ii) The delivery of goods by one person to another for some specific purpose and time is known as: (a) Mortage (b) Pledge (c) Bailment (d) Charge (iii) An agency in which the agent himself has interest in the subject matter of agency is called: (a) Agency by estoppel (b) Agency by holding out (c) Agency by necessity (d) Agency coupled with interest (c) Explain clearly the concept of perpetual-succession and common-seal in relation to a company incorporated under the Companies Act, 1956. (5 Marks) (d) What is the law and procedure relating to registration of a non-profit organization as a company under the Companies Act, 1956 ? (5 Marks) (2 1= 2 Marks) An agreement with a minor may be ratified on his attaining majority.

(II) Choose the correct answer from the following :

INTEGRATED PROFESSIONAL COMPETENCE EXAMINATION : MAY, 2011

Answer (a) Compensation on Breach of Contract: Section 73 of the Indian Contract Act, 1872 provides that when a contract has been broken, the party who suffers by such breach is entitled to receive from the party who has broken the contract, compensation for any loss or damage caused to him thereby which naturally arose in the usual course of things from such breach or which the parties knew when they made the contract, to be likely to result from the breach of it. Such compensation is not given for any remote and indirect loss or damage sustained by reason of the breach. The explanation to the section further provides that in estimating the loss or damage from a breach of contract, the means which existed of remedying the inconvenience caused by the non-performance of the contract must be taken into account. (b) (I) (i) (ii) (II) (i) (ii) Incorrect Incorrect (b) A minor and a major can enter into an agreement of partnership. (c) Bailment.

(iii) (d) Agency coupled with interest (c) Perpetual Succession and Common Seal: A company is a juristic person with a perpetual succession. It never dies nor does its life depends upon the life of its members. It is not in any manner affected by insolvency, mental disorder or retirement of any of its members. It is created by a process of law and can be put to an end only by the process of law. Members may come and go but the company can go on forever (until dissolved). It continues to exist even if all its human members are dead. Since a company had independent existence and since all acts of the company are done in the name of the company, it enjoys a Seal known as common seal. Common seal is equivalent to signature of the company and is affixed on all documents issued by the company. Common seal of the company is kept in safe custody by a responsible officer of the company. (d) Registration of Non-Profit Organisation: An association of persons set up for promoting commerce, art, science, religion, charity or any other useful object and intends to apply its profits or other income in promotion of its objects can be registered as a company under the Companies Act, 1956. However, it has to prohibit payment of any dividend to its members. The association has to apply to the Central Government for issuing a licence. Through this licence the Central Government shall direct the Registrar of Companies to register the association as a company with limited liability without the addition of words limited or private limited to its name. Therefore, the association may be registered accordingly.

26

PAPER 2 : BUSINESS LAWS, ETHICS AND COMMUNICATION

The association has to fulfill the conditions needed for registration as a company, i.e. it must have its name, its Memorandum of Association, its Articles of Association and signatures of its founder members with two witnesses. On registration it will have the same privileges and obligations as a limited company has. Question 2 (a) During the financial year 2010-2011 Mr. Ram was a temporary employee in Ayurved Products Limited and drawing a salary of ` 6000/- per month . On the basis of charge of violent behavior within the premises of the company, he was prevented from working in the company for 60 days pending inquiry. Since there was no adverse conclusion against him, he was reinstated in the service with back salary. He worked for the remaining ten months in that financial year and thereafter resigned from the service. Afterwards, when bonus was paid to others employees, the company refused to pay bonus to Mr. Ram. Decide, whether Mr. Ram will be entitled to bonus under the provisions of the Payments of Bonus Act, 1965? (8 Marks) (b) State the elements which create discrimination in employment in the business organizations. (8 Marks) Answer (a) Payment of Bonus: As per Section 9 of the Payment of Bonus Act, 1965, an employee shall be disqualified from receiving bonus under this Act, if he is dismissed from service for (a) fraud; or (b) riotous or violent behavior while on the premises of the establishment; or (c) theft, misappropriation or sabotage of any property of the establishment. If an employee is guilty of riotous and disorderly behavior he is disqualified for bonus but if such employee because of riotous act, is being prevented from working in the company, pending enquiry and later on he is reinstated as there is no adverse findings, with back wages, will be entitled for bonus [Gannon India Ltd. Vs. Niranjan Das (1984) 2LLJ.223]. In the given case Mr. Ram has worked as a temporary employee for 10 months continuously, so he is qualified and entitled to bonus. Here, a temporary workman is also entitled to bonus on the basis of total number of days which he has completed. After completion of the required period of services for the entitlement to bonus if a workman resigns, he will be entitled to bonus. Therefore, refusal of company is not valid and Mr. Ram will be entitled to the bonus from every angle according to Section 9 of the Payment of Bonus Act, 1965. (b) Elements of Discrimination: Generally, the discrimination means to distinguish one object from another or treating people differently. It is usually intended to refer to the
27

INTEGRATED PROFESSIONAL COMPETENCE EXAMINATION : MAY, 2011

wrongful act of making a difference in treatment or favour on a basis other than individual merit. Such discrimination may also be related in employment in business organization. The elements which create discrimination may be summarized as follows: (i) If the decision against one or more employees is taken which is not based on individual merit, such as the ability to perform a given job, seniority or other morally legitimate qualification. If the decision has been derived solely from racial or sexual prejudice, false stereotypes other kind of morally unjustified attitude against members of which the employee belongs.

(ii)

(iii) If the decision has a harmful or negative impact on the interests of the employees, perhaps costing them jobs, promotions or better pay. Discrimination in employment is wrong because it violates the basic principle of justice by differentiating between people on the basis of characteristics (race or sex) that are not relevant to the tasks they must perform. Looking to these aspects law has also been changed to conform to these moral requirements and to minimize the discrimination in employment in this respect. Question 3 (a) How is the amount of gratuity, payable to employees in a seasonal as well as other establishments, calculated under the provisions of the Payment of Gratuity Act, 1972? What is the maximums amount of gratuity payable under the said Act ? (8 Marks) (b) What are the tips for improving inter-personal skills in a business organization ? (8 Marks) Answer (a) Payment of Gratuity: Section 4 of the Payment of Gratuity Act, 1972 stipulates the manner in which the amount of gratuity payable to an employee will be calculated. In the case of establishments other than seasonal establishments, the employer shall pay the gratuity to an employee at the rate of 15 days wages based on the rate of wages last drawn by the employee concerned for every completed year of service or part thereof in excess of 6 months. In the case of piece rated employees, daily wages shall be computed on the average of the total wages received by him for a period of 3 months immediately preceding the termination of his employment and for this purpose the wages paid for any overtime work shall not be taken into account. In the case of a monthly rated employee 15 days wages shall be calculated by dividing the monthly rate of wages last drawn by 26 and by multiplying the quotient by 15. In the case of seasonal establishment the employees can be classified into two groups (i) Those who work throughout the year and
28

PAPER 2 : BUSINESS LAWS, ETHICS AND COMMUNICATION

(ii)

Those who work only during the season.

The former are entitled to get the gratuity at the rate of 15 days wages for every completed year of service or part thereof in excess of 6 months. The latter are entitled to receive gratuity at the rate of 7 days for each season. As per the Payment of Gratuity (Amendment) Act, 2010, the ceiling on the gratuity amount ` 3,50,000 has been increased to ` 10 lakhs. (b) Tips for improving interpersonal skills: Lines of communication must be open between people who rely on one another to get work done. Poor interpersonal communication skills, which include active listening, result in low productivity simply because one does not have the tools needed to influence, persuade and negotiate which are necessary for workplace success. To get this success the following tips are suggested: (i) Congruency in communication elements: If the words used are incongruent with the other interpersonal communication dynamics interpersonal communication is adversely affected. Since communication is shared meaning, words must send the same message as the other interpersonal communication dynamics body language, facial expression, posture, movement, tone of voice to help emphasize the truth, sincerity and reliability of the communication. A consistent message ensures effective communication. Listening effectively: Effective or active listening is very important skill to enhance interpersonal communication. Listening helps to build strong personal relationships. The process of communication completes when the message as intended by the sender is understood by the receiver. Most of the persons assume that listening is natural trait, but practically very few of us listen properly. One needs to give the communicator of the message sufficient attention and make an effort to understand his view point.

(ii)

Question 4 (a) Whether shares at premiums can be issued by a company ? What are the purposes for which the share premium account can be used under the provisions of the Companies Act, 1956 ? (8 Marks) (b) What are the fundamental principles of ethics applicable to the persons of finance and accounting profession ? (4 Marks) (c) Explain the merits and limitations of oral communication. Answer (a) Share at Premium: If the market exists, a company may issue its shares at premium i.e. the price higher than their normal value. There is no restriction contained in the Companies Act, 1956 on the
29

(4 Marks)

INTEGRATED PROFESSIONAL COMPETENCE EXAMINATION : MAY, 2011

sale of shares at a premium. But SEBI (ICDR) Regulations, 2009 have to be observed as they indicate when an issue has to be at par and when premium is chargeable. Premium may be received in cash or kind, where the value of assets received by a company as a consideration for allotment is greater than the normal value of shares, it is in essence an allotment at a premium. An amount equal to extra value of the assets would have to be carried to the Securities Premium Account. The amount to the credit of share premium account has to be maintained with the same sanctity as share capital and can be reduced only in the manner of share capital. The act does regulate the disbursement of the amount collected as premium. Such account be used in the following ways by the company (a) It may be applied to issue to the members as fully paid by way of bonus the unissued shares of the company. (b) It may be used to write off preliminary expenses. (c) It may be used to write off commission or discount account. (d) It may be spent in providing for the premium payable on the redemption of preference shares or debentures of the company. (b) Principles of Ethics The fundamental principles relating to ethics as applicable to accounting and finance professionals are as follows: (i) The principle of integrity: Integrity means veracity. The principle requires all accounting and finance personnel to be honest and straight-forward in discharging their respective professional duties. The principle of objectivity: The principle requires accounting and financial professionals to stick to their professional and financial judgement without bias, conflicting interests, or under influence of others.

(ii)

(iii) The principle of confidentiality: The principle requires accounting and financial professionals to refrain from disclosing confidential information related to their work. (iv) The principle of professional competence and due care: The financial and accounting professional need to update their professional skill in the modern competitive environment. (v) The principle of professional behavior: The principle requires accounting and financial professional to comply with relevant laws and regulations and avoid such action which may result into discrediting the profession. (c) Oral Communication its merits and limitations - Communication through the spoken word is known as oral communication. Some of the merits of oral communication are as under: (i) saves time and money;
30

PAPER 2 : BUSINESS LAWS, ETHICS AND COMMUNICATION

(ii)

immediate feed back;

(iii) saves paper work; (iv) an effective tool for exhortation; (v) builds a healthy climate; (vi) best tool during emergency. Some of the limitations of Oral Communication are: (i) (ii) Greater chances of misunderstanding; Bad speaker;

(iii) Ineffective for lengthy communication; (iv) Lower retention rate; (v) No legal validity; (vi) Difficult to fix responsibility. Question 5 (a) Point out the differences between a Cheque and a Bill of Exchange under the Negotiable Instruments Act, 1881. (8 Marks) (b) State with reasons whether the following statements are correct or incorrect: (2 2 = 4 Marks) (i) (ii) The responsibility of the corporate management lies towards shareholders only. Creation of proper ethical environment requires a proper understanding of the reasons which lead to an unethical behavior.

(c) Importance of communication is increasing day-by-day in the business organizations. State the reasons for this increasing importance. (4 Marks) Answer (a) Cheque and Bill of Exchange Following are the difference between a cheque and a bill of exchange:1. 2. 3. In cheque, the drawee is always a bank, whereas in a bill of exchange, the drawee may be a bank or any other person. In cheque, days of grace are not allowed, whereas in a bill of exchange, 3 days of grace are allowed for payment. Notice of dishonour is not needed in a cheque, whereas notice of dishonour is usually required in case of a bill of exchange.

31

INTEGRATED PROFESSIONAL COMPETENCE EXAMINATION : MAY, 2011

4. 5.

A cheque can be drawn to bearer and made payable on demand, whereas a bill of exchange cannot be drawn to bearer, if it is made payable on demand. Cheque does not require presentment for acceptance. It needs presentment for payment. Bill of Exchange, sometimes, requires presentment for acceptance and it is advisable to present them for acceptance even when it is not essential to do so. Cheque does not require to be stamped in India, whereas bill of exchange must be stamped according to Law. A cheque may be crossed whereas a bill of exchange cannot be crossed. A cheque being a revocable mandate, the authority may be revoked by counter manding payment, and is determined by notice of the customers death or insolvency. This is not so in a bill. The drawer of a bill of exchange is discharged from liability, if it is not duly presented for payment but the drawer of a cheque is not discharged by delay of the holder in presenting the cheque for payment unless the drawer has suffered some loss due to delay. Incorrect: The traditional governance model positions management is accountable solely to the shareholders only. But a growing number of corporations accept the constituents other than shareholders are affected by corporate activity and that the corporations must, therefore, be responsible to them. These may be enumerated as follows: (a) Employees (b) Trade Unions (c) Customers (d) Shareholders and investors (e) Suppliers (f) Local Communities (g) Government (h) Competitors.

6. 7. 8.

9.

(b) (i)

(ii)

Correct: A creation of a proper ethical environment requires a proper understanding of the reasons which lead to an unethical behavior. The reasons may be summarized as follows: (a) Emphasis on short term results (b) Ignoring small unethical issues (c) Economic Cycles (d) Change in accounting rules.
32

PAPER 2 : BUSINESS LAWS, ETHICS AND COMMUNICATION

(c) Reasons for increasing importance of communication: It is true that importance of communication is increasing day by day in the business organizations. The reasons for this growth may be stated as follows: (a) Growth in the size and multiple locations of organization: Most of the organizations are growing larger and large in size. The people working in these organizations may be spread over different states of a country or over different countries. Keeping in touch, sending directions across and getting feedback is possible only when communication lines are kept working effectively. (b) Growth of trade unions: Over the last so many decades trade unions have been growing strong. No management can be successful without taking the trade unions in confidence. Only through effective communication can a meaningful relationship be built between the management and workers. (c) Growing importance of human relations: Workers in an organization are not like machines. They have their own hopes and aspirations. Management has to recognize them above all as sensitive human beings and work towards a spirit of integration with them which effective communication helps to achieve. (d) Public Relations: Every organization has a social responsibility, towards customers, government, suppliers and the public at large. Communication with them is the only way an organization can project a positive image of itself. (e) Advance in Behavioural Sciences: Modern management is deeply influenced by exciting discoveries made in behavioural sciences like psychology, sociology, transactional analysis etc. All of them throw light on subtle aspects of human nature and help in developing a positive attitude towards life and building up meaningful relationships. And this is possible only through communication. (f) Technological advancement: The world is changing very fast, owing to scientific and technological advancements. These advancements deeply affect not only methods of work but also the composition of groups. In such a situation proper communication between superiors and subordinates becomes very necessary.

Question 6 (a) Describe the procedure for converting a private company into a public company under the provisions of the Companies Act, 1956. (8 Marks) (b) State in brief the guidelines for managing ethics and to prevent the need for whistleblowing in the work place. (4 Marks) (c) What are the characteristics of group personality ? (4 Marks)

33

INTEGRATED PROFESSIONAL COMPETENCE EXAMINATION : MAY, 2011

Answer (a) Conversion of a private company into a public company: The procedure for conversion of a private company into a public company is as follows: (1) Take necessary decision in its board meeting and fix up time, place and agenda for convening Annual General Meeting. (2) Amend Memorandum of Association to change its name by removing the word Private by a special resolution. Approval of the Central Government is not necessary. (3) Pass a Special Resolution deleting from its articles the requirement of a private company under Section 3(1). A copy of the Special Resolution must be filed with the Register of Companies within 30 days. It becomes a public company on the date of alteration. [Section 44(1)]. (4) Increase the number of shareholders/members to at least 7 and number of directors to atleast 3. (5) Within 30 days from the passing of Special Resolution, a prospectus or a statement in lieu of prospectus in the prescribed form must be filed with the Registrar (Section 44). (6) The aforesaid prospectus or the statement in lieu of prospectus must be in conformity with Part I and II of Scheduled II or with Part I and II of Schedule IV respectively. (7) The company has to apply to the Registrar of Companies for the issue of a fresh Certificate of Incorporation, for the changed name, namely, the existing name with the work Private deleted. (b) Managing ethics and preventing whistle-blowing: The focus on core values and sound ethics, the hall mark of ethical management, is being recognized as an important way to ensure the long term effectiveness of governance structures and procedures and to avoid the need for whistle blowing. Employers, who understand the importance of work place ethics, provide their work force with an effective framework and guiding principles of identity and address ethical issues as they arise. These guidelines for managing ethics and to avoid the need for whistleblowing in the work place may be summarized as follows:(a) Have a Code of Conduct and ethics. (b) Establishment of open communication. (c) Make ethical decisions in group and make decision public whenever appropriate. (d) Integrate ethics with other management practices.

34

PAPER 2 : BUSINESS LAWS, ETHICS AND COMMUNICATION

(e) Use of cross functional teams when developing and implementing the ethics management programme. (f) Appointing an ombudsman. (g) Creating an atmosphere of trust. (h) Regularly updating of policies and procedures (i) (j) Include a grievance policy for employees Set an example from the top.

(c) Characteristics of group personality Following are the characteristics of group personality: (a) Spirit of conformity: Individual member soon come to realize that in order to gain recognition, admiration and respect from others they have to achieve a spirit of conformity. Our beliefs, opinions and actions are influenced more by group opinion than by an individuals opinion even if it is an expert opinion. (b) Respect for group values: Any working group is likely to maintain certain values and ideals which make it different from others. In order to deal effectively with a group we must understand its values which will guide us in foreseeing its programmes and actions. (c) Resistance to change: It has been observed that a group generally does not take kindly to social change. On the other hand the group may bring about its own changes, whether by dictation of its leader or by consensus. The degree to which a group resists change serves as an important index of its personality. It helps us in dealing with it efficiently. (d) Group prejudice: Just as hardly any individual is free from prejudice, groups have their own clearly evident prejudice. It is a different matter that the individual members may not admit their prejudiced attitude to others race, religion, nationality etc. But the fact is that the individuals prejudices get their intensified while coming in contact with other members of the group holding similar prejudices. (e) Collective power: Groups are always more powerful than individuals, how so ever influential the individual may be. That is why individuals may find it difficult to speak out their minds in groups. There is always the risk of the one-against many situations cropping up. Question 7 (a) Write a short note on the composition and functions of the Central Board of Trustees under the Employees Provident Fund and Miscellaneous Provisions Act, 1952. (4 Marks) (b) Which documents are required to be filed with the Registrar of Companies at the time of registration of a company under the provisions of the Companies Act, 1956 ? (4 Marks)
35

INTEGRATED PROFESSIONAL COMPETENCE EXAMINATION : MAY, 2011

OR State the conditions whereunder the issuing of prospectus is not necessary under the provisions of the Companies Act, 1956 . (c) State the pressures which are faced by the finance and accounting professionals in an organization in the compliance of fundamental principles of ethics. (4 Marks) OR Write a note on ecological ethics. (d) How is noise a barrier to effective communication ? OR Explain the concept of Negotiation. What are its techniques ? Answer (a) Central Board of Trustees: Under Section 5 of the Employees Provident Funds and Miscellaneous Provisions Act, 1952, the Central Board of Trustees consists of the following: (a) Chairman and Vice-Chairman appointed by the Central Government. (b) Central Provident Fund Commissioner as ex-officio member. (c) More than 5 officials of Central Government. (d) Not more than 15 persons representing the State Government. (e) 10 persons representing employees appointed by the Central Government. (f) 10 persons representing employers appointed by the Central Government. The functions of the Board are as follows: The fund of Employees Provident Funds Scheme (EPF) under Section 5, Employees Pension Scheme under Section 6A and Employees Deposit Linked Insurance Scheme (EDLI) under Section 6C is vested in the Central Board of Trustees. The fund is administered by them as provided in the scheme. The Central Board will perform other functions as may be required under any provisions of PF Scheme, pension scheme and Insurance scheme (Section 5A). (b) Filing of document with the Registrar of Companies: After getting the name approved, the following documents along with the application and prescribed fee, are to be filed with the Registrar:(1) Memorandum of Association [Section 33(1)(a)] (2) Articles of Association, if any [Section 33(1)(b)] (4 Marks)

36

PAPER 2 : BUSINESS LAWS, ETHICS AND COMMUNICATION

(3) The agreement, if any, which the company proposed to enter into with any individual for appointment as its Managing or Whole Time Director or Manager [Section 33(1)(c)]. (4) A declaration that the requirements of the Act and the rules framed there under have been complied with. This declaration is required to be signed by an advocate of the Supreme Court or High Court or an attorney or a pleader having the right to appear before High Court or a Company Secretary or a Chartered Accountant in whole time practice in India who is engaged in the formation of a company, or by person named in the Articles as a Director, Manager or Secretary of the company [Section 33(2)]. (5) In the case of a public company having share capital, where the Articles name a person as director/directors, the list of the directors and their written consent in prescribed form to act as directors and take up qualification shares.(Section 266). (6) Apart from the above, the company must give a notice regarding the situation of its registered office under Section 146 within 30 days of registration. (OR) (b) Non-issuing of Prospectus: As per Section 56 of the Companies Act, 1956, the issue of prospectus is not necessary in the following cases (1) Where shares or debentures are offered to existing holders of shares or debentures. (2) When the issue relates to shares or debentures uniform in all respects, with shares or debentures previously issued and dealt in or quoted in a recognized stock exchange. (3) Where a person is bona-fide invited to enter into an underwriting agreement. (4) Where shares are not offered to the public. (c) Pressures faced by finance and accounting professionals: The finance and accounting professionals are supposed to support the legitimate and ethical objectives established by the employer. As they are having responsibilities to an employing organization, may be under pressure to act or behave in ways that could directly or indirectly threaten compliance with the fundamental principles. Such pressures may be explicit or implicit which may come from supervisor, manager, director or another individuals. Such pressures which are being faced by finance and accounting professionals may be stated as follows: (a) To act contrary to Law or Regulation. (b) To act contrary to technical a professional standards. (c) To facilitate unethical or illegal earnings management strategies.
37

INTEGRATED PROFESSIONAL COMPETENCE EXAMINATION : MAY, 2011

(d) Lie to, or otherwise intentionally mislead other, in particular the auditors of the employing organization or Regulators. (e) To issue or otherwise be associated with, a financial or non financial report that materially misrepresents the facts, including statements, in connection with. For example: (i)
The financial statements

(ii) Tax compliance (iii) Legal compliance, or (iv) Reports required by securities regulators. OR (c) Note on Ecological Ethics: The problem of pollution and other environmental issues can best be framed in terms of our duty to recognize and preserve the ecological systems within which we live. An ecological system is an interrelated and interdependent set of organisms and environments, such as a lake, in which the fish depend on small aquatic organisms, which in turn live off decaying plant and fish waste products. Since the various parts of an ecological system are interrelated, the activities of one of its parts will affect all other parts. Business and all social firms are parts of a larger ecological system. Business firms depend on the natural environment for their energy, material resources, waste disposal and that environment in turn is affected by the commercial activities of business firms. Unless business recognize the interrelationship and interdependencies of the ecological systems within which they operate and unless they ensure that their activities will not seriously injure these systems one cannot hope to deal with the problem of pollution. Ecological ethics is based on the idea that the environment should be protected not only for the sake of human being but also for its own sake. The issue of environmental ethics goes beyond the problem relating to protection of environment or nature in terms of pollution, resource utilization or waste disposal. It is the issue of exploitive human nature and attitudes that should be addressed in a rational way. Problems like global warming, ozone depletion and disposal of hazardous waste that concern the entire world. They require international co-operation and have to be tackled at the global level. (d) Noise as barrier to communication: Noise is the first and foremost barrier to communication. It means interference that occurs in a signal and prevents you from hearing sounds properly. In a factory the continuous noise made by machines makes oral communication difficult. In the same way, same technical problems in a public address system or a static in a telephone or television cable will distort the sound signal and affect communication. Adverse weather

38

PAPER 2 : BUSINESS LAWS, ETHICS AND COMMUNICATION

conditions or some fault in the ultramodern telecommunications systems may also spoil the effect. Further, the sender may resort to ambiguous or confusing signals. The receiver may mess up the message owing to inattention or may spoil decoding because of wrong or unexpected interpretation. The receivers prejudices may also come in the way of his understanding the message in the right spirit. Thus the communication is always likely to affected by noise that stands for so many things. Some of the factors contributing towards noise factors are as follows: (a) Poor Listening: A last moment communication with deadline may put too much pressure on the receiver and may result in resentment. (b) In appropriate Channel: Poor choice of channel of communication can also be contributory to them in understanding of the message. (c) Network breakdown: Some time staff may forget to forward a letter or there may be professional jealousy resulting in closed channel.

OR
(d) Negotiation: Negotiation occurs when two or more parties either individuals or groups discuss specific proposals in order to find a mutually acceptable agreement. Whether it is with an employer, family member or business associate, we all negotiate for things each day like higher salary, letter service or solving a dispute with a co worker or family member. Negotiation is a common way of settling conflicts in business. When handled skillfully, negotiation can improve the position of one or even both but when poorly handled; it can leave a problem still unsolved and perhaps worse than before. Techniques for Negotiation: (a) Spiraling agreements: Begin by reaching a minimums agreement even though it is not related to the objectives and build, hit by hit, on this first agreement. (b) Changing of position: Formulate the proposals in a different way, without changing the final result. (c) Gathering information: Ask for information from the other party to clarify their position (d) Making the cake bigger: Offer alternatives that may be agreeable to the other party, without changing the terms. (e) Commitments: Formalize agreements orally and in writing before ending the negotiation.

39

PAPER 3 : COST ACCOUNTING AND FINANCIAL MANAGEMENT Question No. 1 is compulsory. Attempt any five questions from the remaining six questions. Working Notes should form part of the answer. Question 1 (a) You are given two financial plans of a company which has two financial situations. The detailed information are as under: Installed capacity Actual production and sales Selling price per unit Variable cost per unit Fixed cost: Situation A = ` 20,000 Situation B = ` 25,000 Capital structure of the company is as follows: Financial Plans XY XM 10,000 units 60% of installed capacity

` 30 ` 20

`
Equity Debt (cost of debt 12%) 12,000 40,000 52,000 (b) You are given the following information of a worker: (i) (ii) Name of worker Ticket No. : : : : : : : : X 002 1-4-11 at 8 a.m. 5-4-11 at 12 noon Production of 2,160 units 2000 units 40 units per hour

`
35,000 10,000 45,000

You are required to calculate operating leverage and financial leverage of both the plans.

(iii) Work started (iv) Work finished (v) Work allotted (vi) Work done and approved (vii) Time and units allowed (viii) Wage rate

` 25 per hour

PAPER 3 : COST ACCOUNTING AND FINANCIAL MANAGEMENT

(ix) Bonus (x) (i) (ii) (c) Worker X worked 9 hours a day. Halsey plan and Rowan plan

40% of time saved

You are required to calculate the remuneration of the worker on the following basis:

Prepare a Store Ledger Account from the following transactions of XY Company Ltd. April, 2011 1 5 8 10 15 20 21 22 Opening balance 200 units @ ` 10 per unit. Receipt 250 units costing ` 2,000 Receipt 150 units costing ` 1,275 Issue 100 units Receipt 50 units costing ` 500 Shortage 10 units Receipt 60 units costing ` 540 Issue 400 units

The issues upto 10-4-11 will be priced at LIFO and from 11-4-11 issues will be priced at FIFO. Shortage will be charged as overhead. (d) What is factoring? Enumerate the main advantages of factoring. (4 5 = 20 Marks) Answer (a) Computation of Operating and Financial Leverage Actual Production and Sales: 60% of 10,000 = 6,000 units Contribution per unit: ` 30 ` 20 = ` 10 Total Contribution: 6,000 ` 10 = ` 60,000 Financial Plan Situation Contribution (C) Less: Fixed Cost Operating Profit or EBIT Less: Interest A XY B A XM B

`
60,000 20,000 40,000 4,800
41

`
60,000 25,000 35,000 4,800

`
60,000 20,000 40,000 1,200

`
60,000 25,000 35,000 1,200

INTEGRATED PROFESSIONAL COMPETENCE EXAMINATION : MAY, 2011

Earnings before tax (EBT) Operating Leverage C = EBIT

35,200

30,200

38,800

33,800

60,000 40,000 = 1.5

60,000 35,000 = 1.71 35,000 30,200

60,000 40,000 = 1.5 40,000 38,800 = 1.03

60,000 35,000 = 1.71 35,000 33,800 = 1.04

Financial Leverage =

EBIT EBT

40,000 35,200

= 1.14 = 1.16 (b) No. of units produced and approved = 2,000 Standard time = 40 units per hour Hourly Wage Rate = ` 25 Time allowed = 40 units per hour Time allowed for 2,000 units (i)

2,000 = 50 hours 40
50 hours 40 hours 10 hours = ` 1,000 =` 100
40 10 25 100

Calculation of Remuneration under Halsey Plan: Standard time allowed for 2,000 units : Actual time taken for 2,000 units : Time saved Basic wages for time taken 40 hours @ ` 25 Bonus: 40% of time saved Total

` 1,100

(ii) Calculation of Remuneration Under Rowan Plan: Wages for time taken 40 hours @ ` 25
Bonus = Time saved x (Time Taken x Hourly Rate) Time allowed

= ` 1,000

=
Total

40 10 25 50

= ` 200
` 1,200

42

PAPER 3 : COST ACCOUNTING AND FINANCIAL MANAGEMENT

(c) Store Ledger Account


Name :Code No. :Description:Date Max. Stock Level Min. Stock Level Re-order level Re-order quantityReceipts Issues Qty. Rate Amount Qty. Rate Units Units Bin No.Location Code-

Amount

Qty.

Balance Rate Amount

`
April 1 5 8 250 150 8 8.50

`
2,000 1,275

`
200 200 250 200 250 150

`
10 10 8 10 8 8.50 10 8 8.50 10 8 8.50 10 10 8 8.50 10 10 8 8.50 10 9 8 8.50 10 9

`
2,000 4,000 5,275

10

100

8.50

8.50

15

50

10

500

20

(shortage)

10

10

100

200 250 50 200 250 50 50 190 250 50 50 190 250 50 50 60 40 50 50 60

4,425

4,925

4,825

21

60

540

5,365

22

190 210

10 8

3,580

1,785 (Closing Stock)

43

INTEGRATED PROFESSIONAL COMPETENCE EXAMINATION : MAY, 2011

(d) Concept of Factoring and its Main Advantages: Factoring involves provision of specialized services relating to credit investigation, sales ledger management purchase and collection of debts, credit protection as well as provision of finance against receivables and risk bearing. In factoring, accounts receivables are generally sold to a financial institution (a subsidiary of commercial bank called factor), who charges commission and bears the credit risks associated with the accounts receivables purchased by it. Advantages of Factoring

The main advantages of factoring are: (i) (ii) The firm can convert accounts receivables into cash without bothering about repayment. Factoring ensures a definite pattern of cash inflows.

(iii) Continuous factoring virtually eliminates the need for the credit department. Factoring is gaining popularity as useful source of financing short-term funds requirement of business enterprises because of the inherent advantage of flexibility it affords to the borrowing firm. The seller firm may continue to finance its receivables on a more or less automatic basis. If sales expand or contract it can vary the financing proportionally. (iv) Unlike an unsecured loan, compensating balances are not required in this case. Another advantage consists of relieving the borrowing firm of substantially credit and collection costs and from a considerable part of cash management.
Question 2 (a) You are given the following information of the three machines of a manufacturing department of X Ltd.: Preliminary estimates of expenses Total A (`) Depreciation Spare parts Power Consumable stores Insurance of machinery Indirect labour Building maintenance expenses
44

(per annum) Machines B (`) 7,500 4,000 2,500 C (`) 5,000 2,000 2,500 (`) 7,500 4,000 3,000

20,000 10,000 40,000 8,000 8,000 20,000 20,000

PAPER 3 : COST ACCOUNTING AND FINANCIAL MANAGEMENT

Annual interest on capital outlay Monthly charge for rent and rates Salary of foreman (per month)

50,000 10,000 20,000

20,000

20,000

10,000

Salary of Attendant (per month) 5,000 (The foreman and the attendant control all the three machines and spend equal time on them.) The following additional information is also available: Machines A Estimated Direct Labour Hours Ratio of K.W. Rating 1,00,000 3 B 1,50,000 2 C 1,50,000 3

Floor space (sq. ft.) 40,000 40,000 20,000 There are 12 holidays besides Sundays in the year, of which two were on Saturdays. The manufacturing department works 8 hours in a day but Saturdays are half days. All machines work at 90% capacity throughout the year and 2% is reasonable for breakdown. You are required to : Calculate predetermined machine hour rates for the above machines after taking into consideration the following factors: An increase of 15% in the price of spare parts. An increase of 25% in the consumption of spare parts for machine B & C only. 20% general increase in wages rates.

(b) The Marketing Manager of XY Ltd. is giving a proposal to the Board of Directors of the company that an increase in credit period allowed to customers from the present one month to two months will bring a 25% increase in sales volume in the next year. The following operational data of the company for the current year are taken from the records of the company: ` Selling price Variable cost Total cost Sales value 21 p.u. 14 p.u. 18 p.u. 18,90,000

45

INTEGRATED PROFESSIONAL COMPETENCE EXAMINATION : MAY, 2011

The Board, by forwarding the above proposal and data requests you to give your expert opinion on the adoption of the new credit policy in next year subject to a condition that the companys required rate of return on investments is 40%. (8 + 8 = 16 Marks) Answer (a) Computation of Machine Hour Rate

Basis of apportionment
(A) Standing Charges

Total
`

Machines A
`

B
`

C
`

Insurance Indirect Labour Building Maintenance expenses Rent and Rates Salary foreman Salary attendant of of

Depreciation Basis Direct Labour Floor Space

8,000 24,000 20,000

3,000 6,000 8,000

3,000 9,000 8,000

2,000 9,000 4,000

Floor Space Equal Equal

1,20,000 2,40,000 60,000 4,72,000

48,000 80,000 20,000 1,65,000 84.75

48,000 80,000 20,000 1,68,000 86.29

24,000 80,000 20,000 1,39,000 71.40

Total standing charges Hourly rate for standing charges


(B) Machine Expenses:

Depreciation Spare parts Power Consumable Stores

Direct Final estimates K.W. rating Direct

20,000 13,225 40,000 8,000

7,500 4,600 15,000 3,000

7,500 5,750 10,000 2,500

5,000 2,875 15,000 2,500

46

PAPER 3 : COST ACCOUNTING AND FINANCIAL MANAGEMENT

Total Machine expenses Hourly Rate for Machine expenses Total (A + B) Machine rate Hour

81,225

30,100 15.46

25,750 13.23

25,375 13.03

553,225

1,95,100 100.21

1,93,750 99.52

1,64,375 84.43

Working Notes:

(i)

Calculation of effective working hours: No. of holidays 52 (Sundays) + 12 (other holidays) = 64 Saturday (52 2) = 50 No. of days (Work full time) = 365 64 50 = 251
Hours

Full days work 251 8 = 2,008 Half days work 50 4 = 200 2,208
Hours

Effective capacity 90% of 2,208


Less: Normal loss of time (Breakdown) 2%

1,987 (Rounded off) 40 (Rounded off) 1,947 A


`

Effective running hour

(ii) Amount of spare parts is calculated as under: B


`

C
`

Preliminary estimates Add: Increase in price @ 15% Add: Increase in consumption @ 25% Estimated cost

4,000 600 4,600

4,000 600 4,600 1,150 5,750

2,000 300 2,300 575 2,875

4,600

47

INTEGRATED PROFESSIONAL COMPETENCE EXAMINATION : MAY, 2011

(iii) Amount of Indirect Labour is calculated as under:


`

Preliminary estimates Add: Increase in wages @ 20%

20,000 4,000 24,000

(iv) Interest on capital outlay is a financial matter and, therefore it has been excluded from the cost accounts.
(b) Advise regarding Change in Credit Policy Working Notes:

(1)

Present Sales Value Present Selling Price per unit


Present Sales Volume

` 18,90,000 ` 21

18,90,000 90,000 units 21


= 25% = 90,000 + 25% = 90,000 + 22,500 = 1,12,500 units

Expected increase in Sales Volume


Expected Sales Volume in next year

(2) Present total cost (90,000 18)


Add: Variable cost on additional Sales (22,500 14) Total cost of future sales Average cost per unit

16,20,000 3,15,000 19,35,000 19,35,000 = ` 17.2 1,12,500


`

(3) Cost of Sale (1,12,500 17.2) Average collection period


Average Investment in receivables in the proposed credit policy

19,35,000 = 2 months

19,35,000 2 3,22,500 12

48

PAPER 3 : COST ACCOUNTING AND FINANCIAL MANAGEMENT

(4) Additional Investment in receivables

3,22,500 -

90,000 18 12

= 3,22,5001,35,000 = ` 1,87,500 (5) (6) Contribution from additional sales Return on receivables additional investments in = (21 14) 22,500 = ` 1,57,500
1,57,500 100 1,87,500

= 84% Advise: Since the expected rate of return on additional investment in receivables (84%) is more than the required rate of return (40%), the proposed increase in credit period from one month to two months should be accepted and implemented in the next year.
Question 3 The management of MNP Company Ltd. is planning to expand its business and consults you to prepare an estimated working capital statement. The records of the company reveal the following annual information:

`
Sales Domestic at one months credit Export at three months credit (sales price 10% below domestic price) Materials used (suppliers extend two months credit) Lag in payment of wages month Lag in payment of manufacturing expenses (cash) 1 month Lag in payment of Adm. Expenses 1 month Sales promotion expenses payable quarterly in advance Income tax payable in four instalments of which one falls in the next financial year Rate of gross profit is 20%. Ignore work-in-progress and depreciation. The company keeps one months stock of raw materials and finished goods (each) and believes in keeping ` 2,50,000 available to it including the overdraft limit of ` 75,000 not yet utilized by the company. The management is also of the opinion to make 12% margin for contingencies on computed figure. You are required to prepare the estimated working capital statement for the next year. (16 Marks)
49

24,00,000 10,80,000 9,00,000 7,20,000 10,80,000 2,40,000 1,50,000 2,25,000

INTEGRATED PROFESSIONAL COMPETENCE EXAMINATION : MAY, 2011

Answer Preparation of Statement of Working Capital Requirement for MNP Company Ltd. Estimated Working Capital Statement (A) Current Assets in terms of Cash Costs `

Debtors: Domestic Sales Export Sales

1 19,20,000 12

1,60,000 2,40,000 37,500 75,000 2,40,000 1,75,000 9,27,500


`

3 9,60,000 12 1 9,00,000 12 1 28,80,000 12

Prepaid Sales promotion expenses Stock of Raw materials Stock of finished goods

Cash at Bank and in Hand Total Current Assets


(B) Current Liabilities in terms of Cash Costs

Creditors for: Material Wages

2 9,00,000 12 1 7,20,000 24 1 10,80,000 12 1 2,40,000 12

1,50,000 30,000 90,000 20,000 56,250 3,46,250


`

Manufacturing expenses Administrative expenses Income Tax Payable Total Current Liabilities
(C)

Net Current Assets (A B)


Add: 12% margin for contingencies

5,81,250 69,750 6,51,000

Required Working Capital

50

PAPER 3 : COST ACCOUNTING AND FINANCIAL MANAGEMENT

Working Notes: Cash cost of sales is calculated as under: Domestic Sales Less: Gross profit @ 20% ` 24,00,000 4,80,000 `

19,20,000

Export Sales 10,80,000 100 ` = 12,00,000@10% 90


Question 4

10,80,000
1,20,000 9,60,000 28,80,000

The summarized Balance Sheets of XYZ Limited as at 31st March, 2010 and 2011 are given below: Liabilities Preference capital share 2010 (`) 4,00,000 4,00,000 40,000 2,00,000 2011 Assets (`) 2,00,000 Plant Machinery 6,60,000 Long investment 30,000 Goodwill 1,00,000 Current Assets term 1,20,000 Short investment (less than 2 months) 1,75,000 Cash and Bank 9,00,000 Preliminary expenses 2,10,000 1,80,000 25,75,000 21,20,000 25,75,000 and term 2010 (`) 7,00,000 3,20,000 2011 (`) 8,20,000 4,00,000 30,000

Equity share capital Share premium A/c Capital reserve redemption

9,10,000 11,41,000 50,000 84,000

General reserve

P & L A/c Current liabilities Proposed dividend Provision for tax Additional information:

1,30,000 6,40,000 1,60,000 1,50,000 21,20,000

1,00,000 40,000

80,000 20,000

During the year 2011 the company: (i) Preference share capital was redeemed at a premium of 10% partly out of proceeds issue of 10,000 equity shares of ` 10 each issued at 10% premium and partly out of profits otherwise available for dividends.
51

INTEGRATED PROFESSIONAL COMPETENCE EXAMINATION : MAY, 2011

(ii)

The company purchased plant and machinery for ` 95,000. It also acquired another company stock ` 25,000 and plant and machinery ` 1,05,000 and paid ` 1,60,000 in Equity share capital for the acquisition.

(iii) Foreign exchange loss of ` 1,600 represents loss in value of short-term investment. (iv) The company paid tax of ` 1,40,000. You are required to prepare cash flow statement. Answer Preparation of Statement of Cash Flow for XYZ Limited Cash flow statement as per AS 3 for the year ending 31st March, 2011 (a) Cash flow from Operating Activities (16 Marks)

` 4,45,000 80,000 1,600 20,000 5,46,600 25,000 2,60,000 (2,31,000) 6,00,600 (1,40,000)

Profit before tax (2,75,000 + 1,70,000)


Add: Depreciation on machinery

Foreign exchange loss Preliminary expenses written off


Cash flow before working capital adjustment Add: Stock obtained on acquire

Increase in Current Liabilities


Less: Increase in current assets

Cash flow before tax paid


Less: Tax paid

Cash flow from operating activities


(b) Cash flow from Investing Activities

4,60,600 (95,000) (80,000) 1,10,000 (1,60,000) (2,20,000) (2,70,000) 15,600 1,50,000 1,65,600 (1,75,000)

Purchase of Machinery Purchase of investment


(c) Cash flow from Financing Activities

Issue of shares at premium Payment of dividend Redemption of preference shares at premium


Net increase/decrease in cash and cash equivalent (a+b+c) Cash and cash equivalent at the beginning of the year Cash and cash equivalent at the end of the year

52

PAPER 3 : COST ACCOUNTING AND FINANCIAL MANAGEMENT

Working Notes:

1. To balance b/d To bank To acquired from other 2. To bank To balance c/f

Plant and Machinery Account

`
7,00,000 By depreciation 95,000 1,05,000 By balance c/f 9,00,000
Provision for Tax Account

`
80,000 8,20,000 9,00,000

`
1,40,000 By balance b/d 1,80,000 By P & L 3,20,000 3.
Profit for the year 2011

`
1,50,000 1,70,000 3,20,000

`
P&L Account (1,75,000-1,30,000) Transfer to general reserve (1,20,000+1,00,000 for redemption-opening 2,00,000) Proposed dividend Net Profit 4.
Cash and Cash Equivalent

45,000 20,000 2,10,000 2,75,000

Opening balance + short term investment =1,00,000 + 50,000 = ` 1,50,000 Closing balance = Closing cash + short term investment + foreign exchange loss = 80,000+84,000+1,600=` 1,65,600
Question 5 (a) You are given the following information of the cost department of a manufacturing company:

`
Stores: Opening Balance Purchases
53

12,60,000 67,20,000

INTEGRATED PROFESSIONAL COMPETENCE EXAMINATION : MAY, 2011

Transfer from work-in-progress Issue to work-in-progress Issue to repairs and maintenance Shortage found in stock taking Work-in-progress: Opening Balance Direct wages applied Overhead applied Closing Balance Finished products: Entire output is sold at a profit of 12% on actual cost from work-in-progress. Other information:

33,60,000 67,20,000 8,40,000 2,52,000 25,20,000 25,20,000 90,08,000 15,20,000

`
Wages incurred Overhead incurred Income from Investment Loss on sale of fixed assets Shortage in stock taking is treated as normal loss. You are require to prepare: (i) (ii) Stores control account; Work-in-progress control account; 29,40,000 95,50,000 4,00,000 8,40,000

(iii) Costing Profit and Loss account; (iv) Profit and Loss account and (v) Reconciliation statement (b) What is debt securitization? Explain the basic debt securitization process. (12 + 4 = 16 Marks) Answer (a) Stores Leger Control Account

Dr. ` To Balance b/d 12,60,000 By Work-in-progress control A/c


54

Cr. ` 67,20,000

PAPER 3 : COST ACCOUNTING AND FINANCIAL MANAGEMENT

To General adjustment A/C To Work-in Control A/c

ledger progress

67,20,000 By Overhead control A/c 33,60,000 By Overhead (Shortage) By Balance c/d 1,13,40,000
W.I.P Control A/c

8,40,000 A/c 2,52,000 35,28,000 1,13,40,000

control

Dr. ` To Balance b/d To Stores ledger control A/c To Direct wages Control A/c To Overhead control A/c 25,20,000 By Stores ledger control A/c 67,20,000 By Costing P&L A/c (Cost of 25,20,000 Sales) (Balancing figure) 90,08,000 BY Balance c/d 2,07,68,000
Costing Profit and Loss A/c

Cr. ` 33,60,000 1,58,88,000 15,20,000 2,07,68,000

Dr. ` To W.I.P Control A/c To General ledger Adj. A/c (Profit) 1,58,88,000 By General Ledger 19,06,560 Adj. A/c Cost of sales Add 12%Profit 1,77,94,560
Financial Profit and Loss A/c

Cr. ` `

1,58,88,000 19,06,560

1,77,94,560 1,77,94,560

Dr. ` To opening stock : Stores W.I.P To Purchases To Wages 12,60,000 25,20,000 ` By Sales 37,80,000 By Income from investment 67,20,000 By stock: 29,40,000 Stores W.I.P
55

Cr. ` ` 1,77,94,560 4,00,000

Closing 35,28,000 15,20,000 50,48,000

INTEGRATED PROFESSIONAL COMPETENCE EXAMINATION : MAY, 2011

To Overhead To Loss on sale of fixed assets

95,50,000 By loss 8,40,000 2,38,30,000


Reconciliation Statement

5,87,440

2,38,30,000

Dr. ` Profit as per Cost Accounts Add: Income from investments Less : Loss on sale of fixed assets Under absorption of overheads (working note) Loss as per Financial Accounts
Working Notes: Overhead Control Account

Cr. ` 19,06,560 4,00,000 23,06,560 8,40,000 20,54,000 28,94,000 5,87,440

Dr. ` To General Ledger Adj. A/c To stores Ledger Control A/c 9550000 By W.I.P control A/c 252000 By balance c/d (under absorption of overheads) 8,40,000 4,20,000 1,10,62,000

Cr. ` 90,08,000 20,54,000

To stores ledger control A/c To wages control A/c Indirect wages (` 29,40,000-25,20,000)

1,10,62,000

(b) Debt Securitisation and its Basic Process: It is a method of recyling of funds. It is especially beneficial to financial intermediaries to support the lending volumes. Assets generating steady cash flows are packaged together and against this asset pool, market securities can be issued e.g., housing finance, auto loans, and credit card receivable. Process of Debt securitization

(i)

The Origination Function: A borrower seeks a loan from finance company, bank, etc., the credit worthiness of borrower is evaluated and contract is entered into with repayment schedule structured over the life of the loan.
56

PAPER 3 : COST ACCOUNTING AND FINANCIAL MANAGEMENT

(ii)

The Pooling Function: Similar loans or receivables are clubbed together to create an underlying pool of assets. The pool is transferred in favour of special purpose vehicle (SPV), which acts as a trustee for investors.

(iii) The Securitization Function: SPV will structure and issue securities on the basis of assets pool. The securities carry a coupon and expected maturity, which can be asset based/mortgage based. These are generally sold to investors through merchant bankers. Investors are pension funds, mutual funds and insurance funds. The process of securitization is without resource i.e. investors bear the credit risk or risk of default. Credit enhancement facilities like insurance, letter of credit and guarantees are provided.
Question 6 (a) The management of Z Company Ltd. wants to raise its funds from market to meet out the financial demands of its long-term projects. The company has various combination of proposals to raise its funds. You are given the following proposals of the company: (i) Proposals P Q (ii) R Cost of debt 10% Cost of preference shares 10% (iii) Tax rate 50% (iv) Equity shares of the face value of ` 10 each will be issued at a premium of ` 10 per share. (v) Total investment to be raised ` 40,00,000. (vi) Expected earnings before interest and tax ` 18,00,000. From the above proposals the management wants to take advice from you for appropriate plan after computing the following: % of Equity 100 50 50 % of Debts 50 % of Preference shares 50

Earnings per share Financial break-even-point Compute the EBIT range among the plans for indifference. Also indicate if any of the plans dominate. (12 + 4 = 16 Marks)

(b) Distinguish between cost units and cost centres.

57

INTEGRATED PROFESSIONAL COMPETENCE EXAMINATION : MAY, 2011

Answer (a) (i) Computation of Earnings per Share (EPS) Plans P Q R

` Earnings before interest & tax (EBIT)


Less: Interest charges

` 18,00,000 2,00,000 16,00,000 8,00,000 8,00,000 8,00,000 1,00,000


8

` 18,00,000 18,00,000 9,00,000 9,00,000 2,00,000 7,00,000 1,00,000


7

18,00,000 18,00,000 9,00,000 9,00,000 9,00,000 2,00,000


4.5

Earnings before tax (EBT)


Less : Tax @ 50%

Earnings after tax (EAT)


Less : Preference share dividend

Earnings available for equity shareholders No. of shares


E.P.S (Rs.) (ii) Computation of Financial Break-even Points

Proposal P Proposal Q Proposal R

=0 = ` 2,00,000 (Interest charges) = Earnings required for payment of preference share dividend i.e. ` 2,00,000 0.5 (Tax Rate) = ` 4,00,000

(iii) Computation of Indifference Point between the Proposals

The indifference point

(EBIT 11 )(1 T ) (EBIT 12 )(1 T ) = E1 E2 Where, EBIT = Earnings before interest and tax 11 12 T E1 E2 = Fixed Charges (Interest) under Proposal P = Fixed charges (Interest) under Proposal Q = Tax Rate = Number of Equity shares in Proposal P = Number of Equity shares in Proposal Q

58

PAPER 3 : COST ACCOUNTING AND FINANCIAL MANAGEMENT

Combination of Proposals (a) Indifference point where EBIT of proposal P and proposal Q is equal

(EBIT 0)(1 .5) (EBIT 2,00,000)(1 0.5) = 2,00,000 1,00,000 .5 EBIT (1,00,000) = (.5 EBIT -1,00,000) 2,00,000 .5 EBIT = EBIT 2,00,000 EBIT = ` 4,00,000
(b) Indifference point where EBIT of proposal P and Proposal R is equal:

(EBIT 1)(1 T) = E1

EBIT 12)(1 T) - Preference share dividend E2

(EBIT 0)(1 .5) (EBIT 0)(1 .5) 2,00,000 = 1,00,000 2,00,000 .5EBIT .5EBIT 2,00,000 = 2,00,000 1,00,000 .25 EBIT = 0.5 EBIT -2,00,000 EBIT = 2,00,000 0.25 = ` 8,00,000
(c) Indifference point where EBIT of proposal Q and proposal R are equal

(EBIT 2,00,000)(1 0.5) (EBIT 0)(1 0.5) 2,00,000 = 1,00,000 1,00,000 .5 EBIT -1,00,000 = .5 EBIT 2,00,000 There is no indifference point between proposal Q and proposal R
Analysis: It can be seen that Financial proposal Q dominates proposal R, since the financial break-even-point of the former is only ` 2,00,000 but in case of latter, it is ` 4,00,000. (b) Cost units: It is a unit of product, service or time (or combination of these) in relation to which costs may be ascertained or express. A batch which consists of a group of identical items and maintain its identity through one or more stages of production may also be considered as a cost unit. Cost units are usually the units of physical measurement like number, weight, area, volume, length, time and value.
59

INTEGRATED PROFESSIONAL COMPETENCE EXAMINATION : MAY, 2011

Cost centre: It is defined as a location, person or an stress of equipment (or group of these) for which cost may be ascertained and used for the purpose of cost control. Cost centres are of two types, viz., personal and impersonal. A personal cost centre consists of a person or group of persons and an impersonal cost centre consists of a location or an item of equipment (or group of these). Question 7 Answer any four of the following: (a) How do you deal with the following in cost account? (i) (ii) (i) (ii) Packing Expenses Fringe benefits Operating ratio Price earnings ratio

(b) Explain the following ratios:

(c) Enumerate the causes of labour turnover. (d) Write short note on William J. Baumal Vs. Miller-Orr cash management model. (e) Discuss the process of estimating profit/loss on incomplete contracts. (4 4 = 16 Marks) Answer (a) Packing expenses: Cost of primary packing necessary for protecting the product or for convenient handling, should become a part of the prime cost. The cost of packing to facilitate the transportation of the product from the factory to the customer should become a part of the distribution cost. If the cost of special packing is at the request of the customer, the same should be charged to the specific work order or the job. The cost of fancy packing necessary to attract customers is an advertising expenditure. Hence, it is to be treated as a selling overhead. Fringe benefits: These are the additional payments or facilities provided to the workers apart from their salary and direct cost-allowances like house rent and city compensatory allowances. If the amount of fringe benefit is considerably large, it may be recovered as direct charge by means of a supplementary wage or labour rate; otherwise these may be collected as part of production overheads. (b) (i) Concept of Operating Ratio

Operating ratio=

Cost of goods sold + operating expenses 100 Net sales

This is the test of the operational efficiency with which the business is being carried; the operating ratio should be low enough to leave a portion of sales to give a fair return to the investors.
60

PAPER 3 : COST ACCOUNTING AND FINANCIAL MANAGEMENT

(ii)

Concept of Price-Earning ratio

Price Earning Ratio =

Market price per equity share Earning per share

This ratio indicates the number of times the earnings per share is covered by its market price. It indicates the expectation of equity investors about the earnings of the firm.
(c) Causes of Labour Turnover : The main causes of labour turnover in an organisation/industry can be broadly classified under the following three heads :

(a) Personal Causes; (b) Unavoidable Causes; and (c) Avoidable Causes.
Personal causes are those which induce or compel workers to leave their jobs; such causes include the following :

(i) (ii)

Change of jobs for betterment. Premature retirement due to ill health or old age.

(iii) Domestic problems and family responsibilities. (iv) Discontent over the jobs and working environment.
Unavoidable causes are those under which it becomes obligatory on the part of management to ask one or more of their employees to leave the organisation; such causes are summed up as listed below:

(i) (ii)

Seasonal nature of the business; Shortage of raw material, power, slack market for the product etc.;

(iii) Change in the plant location; (iv) Disability, making a worker unfit for work; (v) Disciplinary measures; (vi) Marriage (generally in the case of women).
Avoidable causes are those which require the attention of management on a continuous basis so as to keep the labour turnover ratio as low as possible. The main causes under this case are indicated below :

(i) (ii)

Dissatisfaction with job, remuneration, hours of work, working conditions, etc., Strained relationship with management, supervisors or fellow workers;
61

INTEGRATED PROFESSIONAL COMPETENCE EXAMINATION : MAY, 2011

(iii) Lack of training facilities and promotional avenues; (iv) Lack of recreational and medical facilities; (v) Low wages and allowances. (d) William J Baumal vs Miller- Orr Cash Management Model: According to William J Baumals Economic order quantity model optimum cash level is that level of cash where the carrying costs and transactions costs are the minimum. The carrying costs refer to the cost of holding cash, namely, the interest foregone on marketable securities. The transaction costs refer to the cost involved in getting the marketable securities converted into cash. This happens when the firm falls short of cash and has to sell the securities resulting in clerical, brokerage, registration and other costs. The optimim cash balance according to this model will be that point where these two costs are equal. The formula for determining optimum cash balance is : C =

2UP S C U P S

Where, = Optimum cash balance = Annual (monthly) cash disbursements = Fixed cost per transaction = Opportunity cost of one rupee p.a. (or p.m)

Miller-Orr cash management model is a net cash flow stochastic model. This model is designed to determine the time and size of transfers between an investment account and cash account. In this model control limits are set for cash balances. These limits may consist of h as upper limit, z as the return point, and zero as the lower limit. When the cash balances reach the upper limit, the transfer of cash equal to h-z is invested in marketable securities account. When it touches the lower limit, a transfer from marketable securities account to cash account is made. During the period when cash balance stays between (h,z) and (z, o ) i.e high and low limits no transactions between cash and marketable securities account is made. The high and low limits of cash balance are set up on the basis of fixed cost associated with the securities transactions, the opportunity cost of holding cash and the degree of likely fluctuations in cash balances. These limits satisfy the demands for cash at the lowest possible total costs.
(e) Process of Estimating Profit/Loss on Incomplete Contracts: To determine the profit to be taken to Profit and Loss Account, in the case of incomplete contracts, the following process is followed:

(i)

Completion of contract is less than 25 per cent: In this case no profit should be taken to profit and loss account.
62

PAPER 3 : COST ACCOUNTING AND FINANCIAL MANAGEMENT

(ii)

Completion of contract is upto 25 per cent or more than 25 per cent but less than 50 per cent: In this case one-third of the notional profit, reduced in the ratio of cash received to work certified, should be transferred to the Profit and Loss Account. Mathematically:

1 Cash received Notional Profit 3 Work received


(iii) Completion of contract is upto 50 per cent or more than 50 per cent but less than 90 per cent: In this case, two-third of the notional profit, reduced by proportion of cash received to work certified, is transferred to the Profit and Loss Account. Mathematically :

2 Cash received Notional Profit 3 Work received


(iv) Completion of contract is upto 90 per cent or more than 90 per cent i.e. it is nearing completion: In this case the profit to be taken to Profit and Loss Account is determined by determining the estimated Profit and using any one of the following formulas : (a) Estimated Profit (b) Estimated Profit
Work certified Contract price Work certified Cash received Contract price Work certified

OR Estimated Profit (c) Estimated Profit (d) Estimated Profit (e) Notional Profit
Cash received Contract price

Cost of work to date Estimated total cost Cost of work to date Cash received Estimated total cost Work certified

Work certified Contract price

63

PAPER 4 : TAXATION Question No.1 is compulsory. Attempt any five questions from the remaining six questions. Wherever required, suitable assumptions may be made by the candidate and stated clearly in the answer. Working notes should form part of the answer. All questions relate to the assessment year 2011-12, unless stated otherwise in the question. Question 1 (a) On 21-3-2010, Mr. Janak gifted to his wife Mrs. Thilagam 200 listed shares, which had been bought by him on 19-4-2009 at ` 2,000 per share. On 1-6-2010, bonus shares were allotted in the ratio of 1:1. All these shares were sold by Mrs. Thilagam as under: Date sale of Manner of sale Sold in recognized stock exchange, STT paid Private sale to an outsider Private sale to her friend Mrs. Hema (Market value on this date was ` 2,10,000) No. of Net sales shares value (`) 100 All bonus shares 100 2,20,000 1,25,000 1,70,000

21.5.2010 21.7.2010 28.2.2011

Briefly state the income-tax consequences in respect of the sale of the shares by Mrs. Thilagam, showing clearly the person in whose hands the same is chargeable, the quantum and the head of income in respect of the above transactions. Detailed computation of total income is NOT required. Net sales value represents the amount credited after all taxes, levies, brokerage, etc., and the same may be adopted for computing the capital gains. Cost inflation index for the FY 2010-11 is 711 and for the FY 2009-10 is 632. (5 marks) (b) Nathan Aviation Ltd. is running two industrial undertakings, one in a SEZ (Unit S) and another in a normal area (Unit N). The brief summarized details for the year ended 31.3.2011 are as under: The Suggested Answers for Paper 4: Taxation are based on the provisions of law as amended by the Finance Act, 2010 and applicable for A.Y. 2011-12 (in the case of Income-tax), which is the assessment year relevant for May, 2011 examination.

PAPER 4 : TAXATION

(` in lacs) S Domestic turnover Export turnover Gross profit Less: Expenses and depreciation Profits derived from the unit 10 120 20 7 13 N 100 Nil 10 6 4

(c)

The brought forward business loss pertaining to Unit N is ` 2 lacs. Briefly compute the business income of the assessee. (5 Marks) Pareesh & Co., is a partnership firm engaged in the business of recruitment and supply of labourers. The firm, which had rendered taxable services to the tune of ` 20.2 lacs in the financial year 2009-10, furnishes the following details pertaining to the half year ended on 30.9.2010 : (` ) (i) (ii) Amounts collected from companies for pre-recruitment screening Amounts collected from companies for recruitment of Permanent staff Temporary staff (iii) 3,00,000 4,00,000 2,50,000

Advances received from prospective employers for conducting campus 1,00,000 interviews in colleges Wherever applicable, service tax has been charged separately and received from clients. Compute the value of taxable services rendered and the service tax payable by the assessee for the relevant half year. (5 Marks) (d) The following are details of purchases, sales, etc. effected by Vasudha & Co., a registered dealer, for the year ended 31.3.2011 : Particulars Purchase of raw materials within State, 1000 units, inclusive of VAT levy at 6% Inter-State purchases of raw materials, inclusive of CST at 2% Import of raw materials, inclusive of customs duty of ` 35,000 Capital goods purchased on 1.5.2010, inclusive of VAT levy at 10% (input credit to be spread over 2 financial years) Amount (` ) 5,30,000 2,04,000 4,35,000 3,30,000

65

INTEGRATED PROFESSIONAL COMPETENCE EXAMINATION : MAY, 2011

Other manufacturing expenses Sale of taxable goods within State, inclusive of VAT leyy at 4% Sale of goods within State, exempt from levy of VAT (Goods were manufactured from the Inter-State purchase of raw materials) Closing stock as on 31.3.2011 was 100 units of raw materials purchased within the State

1,50,000 7,28,000 1,20,000

Input credit is allowed only on raw materials used in manufacture of the taxable goods. Compute the VAT liability of the dealer for the year ended 31.3.2011. (5 Marks) Answer (a) Where an asset has been transferred by an individual to his spouse otherwise than for adequate consideration, the income arising from the sale of the said asset by the spouse will be clubbed in the hands of the individual. Where there is any accretion to the asset transferred, income arising to the transferee from such accretion will not be clubbed. Hence, the profit from sale of bonus shares allotted to Mrs. Thilagam will be chargeable to tax in the hands of Mrs. Thilagam. Therefore, the capital gains arising from the sale of the original shares has to be included in the hands of Mr. Janak, and the capital gains arising from the sale of bonus shares would be taxable in the hands of Mrs. Thilagam. Where an asset received by way of gift has been sold, the period of holding of the previous owner should be considered for determining whether the capital gain is long term or short term. The cost to the previous owner has to be taken as the cost of acquisition. Income/loss to be clubbed in the hands of Mr. Janak Long-term capital gains/loss Particulars (i) 100 shares sold on 21.5.2010 in a recognized stock exchange, STT paid. Long-term capital gains on sale of such shares is exempt under section 10(38) Shares sold to a friend on 28.2.2011 Sale consideration Less: Indexed cost of acquisition of 100 shares (` 2,000 x 100 x 711/632) Long term capital loss to be included in the hands of Mr. Janak 1,70,000 2,25,000 (55,000) ` Nil

(ii)

66

PAPER 4 : TAXATION

Income taxable in the hands of Mrs. Thilagam Short-term capital gains (on sale of 100 bonus shares) Particulars Sale consideration Less: Cost of acquisition of bonus shares Short-term capital gains ` 1,25,000 Nil 1,25,000

Taxability in the hands of Mrs. Hema under the head Income from other sources Mrs. Hema has received shares from her friend, Mrs. Thilagam, for inadequate consideration. Even though shares fall within the definition of property under section 56(2)(vii), the provisions of section 56(2)(vii) would not be attracted in the hands of Mrs. Hema, since the difference between the fair market value of shares and actual sale consideration does not exceed Rs.50,000. (b) Computation of business income of Nathan Aviation Ltd. Particulars Total profit dervied from Units S & N (Rs.13 lacs + Rs.4 lacs) Less: Less: Exemption under section 10AA [See Working Note below] Brought forward business loss ` in lacs 17 12 5 2 3 Working Note Computation of exemption under section 10AA in respect of Unit S located in a SEZ Domestic turnover of Unit S Export turnover of Unit S Total turnover of Unit S Profit derived from Unit S Exemption under section 10AA Profit of Unit S x ` in lacs 10 120 130 13

Export turnover of unit S 120 = 13 = Total turnover of Unit S 130

12

67

INTEGRATED PROFESSIONAL COMPETENCE EXAMINATION : MAY, 2011

Note 100% of the profit derived from export of articles or things or from services is eligible for deduction under section 10AA, assuming that F.Y.2010-11 falls within the first five year period commencing from the year of manufacture or production of articles or things or provision of services by the Unit in SEZ. (c) Computation of value of taxable services and service tax payable by M/s Pareesh & Co.:Particulars Taxable services (`) Service tax (`)

Amounts collected from companies for prerecruitment screening Amounts collected from companies recruitment of permanent staff for

2,50,000 3,00,000 4,00,000

25,750 30,900 41,200

Amounts collected from companies for recruitment of temporary staff (It is also a taxable service) Advances received from prospective employers for conducting campus interview in colleges (Note 1)
Total Notes:

1,00,000
10,50,000

10,300
1,08,150

1.

Since the question specifically provides that wherever applicable, service tax has been charged separately, advance has also been taken as exclusive of service tax. However, it may be noted that in actual practice, advance for a taxable service is generally inclusive of service tax and accordingly service tax is computed by making back calculations. In such a case, the amount of service tax and the value of taxable service in relation to the advance received for conducting campus interview would be ` 9,338 and ` 90,662 respectively. Therefore, the total service tax payable and total value of taxable service would be ` 1,07,188 and ` 10,40,662 respectively.

2.

Since the value of taxable services rendered in the preceding year is more than ` 10 lakh (i.e. ` 20.2 lakh), the assessee is not a small service provider. Hence, it is not eligible for the exemption available to the small service provider.
Amount (`)

(d) Computation of VAT liability of Vasudha & Co. for the year ended 31.3.2011:Particulars Input tax credit: Intra-State purchases of 1000 units of raw materials [Refer Note 1]
68

30,000

PAPER 4 : TAXATION

Inter-State purchases of raw materials [Refer Note 2] Import of raw materials [Refer Note 3] Purchase of Capital Goods [Refer Note 4] Other manufacturing expenses [Refer Note 5] Total input tax credit available (A): Output VAT payable: Sale of taxable goods within State [(7,28,000 x 4)/104] Sale of exempted goods within State [Refer Note 6] Total VAT payable (B) Net VAT liability (VAT credit to be carried forward) [(B) - (A)] Notes:1. 2. 3. 4.

15,000 45,000 28,000 28,000 (17000)

5,30,000 6 VAT paid on intra-State purchases is eligible for input tax credit . 106

CST paid on intra-State purchases is not eligible for input tax credit. Customs duty is not eligible for input tax credit. VAT paid on purchase of capital goods is eligible for input tax credit. However, the 3,30,000 10 same has to be spread over a period of two years . 110 2

5. 6.

No input tax credit can be availed on expenses incurred on manufacturing. No VAT will be payable on sale of goods exempted from levy of VAT. Further, since these goods are manufactured from the inter-State purchases of raw materials (nonvattable inputs), input tax credit is not affected. VAT system allows credit in respect of purchases made during a period to be set-off against the taxable sales during that period, irrespective of when the supplies/inputs purchased are utilized/sold. Therefore, input tax credit in respect of closing stock of raw materials need not be reduced from total input tax credit available.

7.

Note: The statement in the question, Input credit is allowed only on raw materials used in manufacture of the taxable goods, implies that the same is not allowable in respect of sale of goods within the State which are exempt from levy of VAT. Question 2 (a) (i) Miss Vivitha paid a sum of 5000 USD to Mr. Kulasekhara, a management consultant practising in Colombo, specializing in project financing. The payment was made in Colombo. Mr. Kulasekhara is a non-resident. The consultancy is related to a project in India with possible Ceylonese collaboration. Is this payment chargeable to tax in India in the hands of Mr. Kulasekhara, since the services were used in India? (4 Marks)

69

INTEGRATED PROFESSIONAL COMPETENCE EXAMINATION : MAY, 2011

(ii)

Mr. Praveen Kumar has furnished the following particulars relating to payments made towards scientific research for the year ended 31.3.2011 : (` in lacs) (i) (ii) (iii) Payments made to K Research Ltd. Payment made to LMN College Payment made to OPQ College Note: K Research Ltd. and LMN College are approved research institutions and these payments are to be used for the purposes of scientific research. (iv) (v) (vi) Payment made to National Laboratory Machinery purchased for in-house scientific research Salaries to research staff engaged in in-house scientific research 8 25 12 20 15 10

Compute the amount of deduction available under section 35 of the Income-tax Act, 1961 while arriving at the business income of the assessee. (4 Marks) (b) During the year ended 31.3.2010, Kohli & Co., running a coaching centre, has collected a sum of ` 10.2 lacs as service tax. ` 70,000 was met through Cenvat credit and the balance was paid by cheques on various dates. The details pertaining to the quarter ended 30.6.2010 are as under: Particulars Value of free coaching rendered Coaching fees collected from students (Service tax collected separately) Advance received from a college for coaching their students, on 30.6.2010. However, no coaching was conducted and the money was returned on 12.4.2011 Amount (` ) 20,000 14,50,000 3,00,000

Determine the service tax liability for the quarter and indicate the date by which the service tax has to be remitted by the assessee. (4 Marks) (c) Which variant of VAT is most widely used in the world and why? Are some services also included in the VAT net by such countries? (4 Marks) Answer (a) (i)

A non-resident is chargeable to tax in respect of income received outside India only if such income accrues or arises or is deemed to accrue or arise to him in India.

70

PAPER 4 : TAXATION

The income deemed to accrue or arise in India under section 9 comprises, inter alia, income by way of fees for technical services, which includes any consideration for rendering of any managerial, technical or consultancy services. Therefore, payment to a management consultant relating to project financing is covered within the scope of fees for technical services. The Explanation below section 9(2) has been substituted to clarify that income by way of, inter alia, fees for technical services, from services utilized in India would be deemed to accrue or arise in India in case of a non-resident and be included in his total income, whether or not such services were rendered in India. In the instant case, since the services were utilized in India, the payment received by Mr. Kulasekhara, a non-resident, in Colombo is chargeable to tax in his hands in India, as it is deemed to accrue or arise in India.
(ii) Computation of deduction allowable under section 35 Particulars Payment for scientific research Amount (` in lacs) Section % of Amount of weighted deduction deduction (` in lacs)

K Research Ltd. [See Note 3] LMN College OPQ College [See Note 1] National Laboratory
In-house research [See Note 2]

20 15 10 8 25 12

35(1)(ii) 35(1)(ii) 35(2AA) 35(1)(iv) r.w. 35(2) 35(1)(i)

175% 175% Nil 175% 100% 100%

35.00 26.25 Nil 14.00 25.00 12.00


112.25

Capital expenditure Revenue expenditure


Deduction allowable under section 35 Notes :1. Payment to OPQ College

Since the note in the question below item (iii) clearly mentions that only K Research Ltd. and LMN College (mentioned in item (i) and (ii), respectively) are approved research institutions, it is a logical conclusion that OPQ College mentioned in item (iii) is not an approved research institution. Therefore, payment to OPQ College would not qualify for deduction under section 35.
2. Deduction for in-house research and development

Only company assessees are entitled to weighted deduction@200% under


71

INTEGRATED PROFESSIONAL COMPETENCE EXAMINATION : MAY, 2011

section 35(2AB) in respect of in-house research and development. However, in this case, the assessee is an individual. Therefore, he would be entitled to deduction@100% of the revenue expenditure incurred under section 35(1)(i) and 100% of the capital expenditure incurred under section 35(1)(iv) read with section 35(2), assuming that such expenditure is laid out or expended on scientific research related to his business.
3. Payment to K Research Ltd. (Alternative Answer)

Any sum paid to a company registered in India which has as its main object scientific research, as is approved by the prescribed authority, qualifies for a weighted deduction of 125% under section 35(1)(iia). Therefore, it is also possible to take a view that payment of ` 20 lakhs to K Research Ltd. qualifies for a weighted deduction of 125% under section 35(1)(iia) since K Research Ltd. is a company. The weighted deduction under section 35(1)(iia) would be ` 25 lacs (i.e., 125% of ` 20 lacs), in which case, the total deduction under section 35 would be ` 102.25 lacs.
(b) Particulars Computation of service tax liability of Kohli & Co. for the quarter ended 30.06.2010:Amount of service tax (`)

Free coaching rendered


14,50,000 10.3 Coaching fees collected from students 100

Nil 1,49,350 28,015


1,77,365

Advance received from a college for coaching their students


Total service tax liability for the quarter ended 30.06.2010 Notes:

1. 2. 3.

Free coaching is not exigible to service tax Coaching fees collected from students will be liable to service tax @ 10.3%. Advance receipt is chargeable to service tax. It is immaterial that no coaching was conducted and the money was returned on 12.4.2011. Advance would be chargeable to service tax in the quarter ended 30.06.2010. Advance has been 3,00,000 10.3 assumed to be inclusive of service tax 110.3 During the preceding financial year, the service tax liability met by the assessee, inclusive of CENVAT credit availed was more than ` 10 lakh. Hence, during the
72

PAPER 4 : TAXATION

current financial year, for all quarters, payment of service tax will have to be made electronically. Therefore, the last date for making the payment of service tax by Kohli & Co. (non-corporate assessee) for the quarter ended 30.06.2010 is 6th July, 2010.
(c) Among the three variants of VAT, the consumption variant is widely used. Several countries of Europe and other countries have adopted this variant. The reasons for preference of this variant are:

Firstly, it does not affect decisions regarding investment because the tax on capital goods is also set-off against the VAT liability. Hence, the system is tax neutral in respect of techniques of production (labour or capital-intensive). Secondly, the consumption variant is convenient from the point of administrative expediency as it simplifies tax administration by obviating the need to distinguish between purchases of intermediate and capital goods on the one hand and consumption goods on the other hand. In practice, therefore, most countries use the consumption variant. Also, most VAT countries include many services in the tax base. Since the business gets set-off for the tax on services, it does not cause any cascading effect.
Question 3 (a) Mr. Vidyasagar, a resident individual aged 64, is a partner in Oscar Musicals & Co., a partnership firm. He also runs a wholesale business in medical products. The following details are made available for the year ended 31.3.2011 : ` (i) (ii) (iii) (iv) Interest on capital received from Oscar Musicals & Co., at 15% Interest from bank on fixed deposit (Net of TDS ` 1,50,000 13,500 34,500 5,60,000 34,000 40,000 7,000

` 1,500)
Income-tax refund received relating to assessment year 2009-10 including interest of ` 2,300 Net profit from wholesale business Amounts debited include the following: Depreciation as per books Motor car expenses Municipal taxes for the shop (For two half years; payment for one half year made on 12.6.2011 and for the other, on 14.11.2011) Salary to manager for whom single cash
73

21,000

INTEGRATED PROFESSIONAL COMPETENCE EXAMINATION : MAY, 2011

payment was made for (v) The WDV of the assets (as on 1.4.2010) used in above wholesale business is as under: Computers Motor car (20% used for personal use) (vi) LIP paid for major son PPF of his wife 1,20,000 3,20,000 60,000 70,000

Long-term infrastructure bonds (Approved) 30,000 Compute the total income of the assessee for the assessment year 2011-12. The computation should show the proper heads of income. Also compute the WDV of the different blocks of assets as on 31.3.2011. (8 Marks) (b) (i) Where any transaction of taxable service is entered into with an associated enterprise, receipt of service tax is not material for levy of service tax. Explain with reasons, (2 Marks) whether you agree or disagree with this statement.

(ii) Briefly discuss about the adjustment of excess amount of service tax paid in case of renting of immovable property service, owing to property tax payment. (2 Marks) (c) M/s. Staruss & Co., a registered dealer under the local VAT law, having stock of goods purchased from outside the State, wishes to opt for the Composition Scheme. Advise him whether the same is possible. Will the VAT chain be broken if the dealer opts for the said scheme? (4 Marks) Answer (a) Computation of total income of Mr. Vidyasagar for the A.Y.2011-12 Particulars Profits and gains of business or profession Income from own business Net profit as per books Add: Depreciation as per books Disallowance of municipal taxes paid for the second half-year under section 43B, since the same was paid after the due date of filing of return (` 7,000/2) Disallowance under section 40A(3) in respect of salary paid in cash since the same exceeds ` 20,000 20% of car expenses for personal use Less: ` `

5,60,000 34,000 3,500 21,000 8,000 66,500 6,26,500 1,10,400 5,16,100

Depreciation allowable (Note 1)

74

PAPER 4 : TAXATION

Income from firm

Interest on capital from partnership firm (Note 2)


Income from other sources

1,20,000
6,36,100

Interest on bank fixed deposit (Gross) Interest on income-tax refund


Gross total income Less:

15,000 2,300 17,300


6,53,400

Deductions under Chapter VIA (Note 3)

1,20,000
5,33,400

Total Income Notes:

(1) Depreciation allowable under the Income-tax Rules, 1962


Opening WDV Block 1 Block 2 Computers Motor Car 1,20,000 3,20,000 Rate 60% 15% 48,000 9,600 38,400 1,10,400 2,81,600 Depreciation 72,000 Closing WDV 48,000

Less: 20% disallowance for personal use

(2) Only to the extent the interest is allowed as deduction in the hands of the firm, the same is includible as business income in the hands of the partner. Maximum interest allowable as deduction in the hands of the firm is 12% p.a. It is assumed that the partnership deed provides for the same and hence is allowable to this extent in the hands of the firm. Therefore, interest @12% p.a. amounting to ` 1,20,000 would be treated as the business income of Mr. Vidyasagar. (3) Deduction under Chapter VI-A
Particulars Under section 80C ` `

LIP for major son PPF paid in wifes name As per section 80CCE, deduction is restricted to
Under section 80CCF

60,000 70,000 1,30,000 1,00,000

Approved Infrastructure bonds (` 30,000 but restricted to


75

INTEGRATED PROFESSIONAL COMPETENCE EXAMINATION : MAY, 2011

` 20,000, being the maximum deduction allowable u/s 80CCF) (b) (i)

20,000

Total deduction 1,20,000 Yes, I agree with the statement. Section 68 casts the liability to pay service tax upon the person liable to pay service tax. This liability is not contingent upon the service provider realizing or charging the service tax rate at the prevailing rate. The statutory liability does not get extinguished if the service provider fails to realize or charge service tax from the service receiver.

It may be noted that in case of transactions of taxable services involving any associated enterprise service tax is liable to be paid even if the value of taxable service has not been received. This is so because in such a case any payment received towards the value of taxable service includes any amount credited or debited to any account, whether called Suspense account or by any other name, in the books of account of a person liable to pay service tax. Thus, service tax would be payable on such credits as well.
(ii) In case of renting of immovable property service, a deduction of property taxes paid in respect of the immovable property is allowed from the gross amount charged for renting of the said immovable property vide Notification No.27/2007 ST dated 22.05.2007. However, where any amount in excess of the amount required to be paid towards service tax liability has been paid on account of non-availment of such deduction, such excess amount may be adjusted against the service tax liability within one year from the date of payment of such property tax.

The details of such adjustment have to be intimated to the Superintendent of Central Excise having jurisdiction over the service provider within a period of 15 days from the date of such adjustment.
(c) As per the principles laid down in the White Paper, a dealer desirous of availing the benefits of VAT Composition Scheme should not have stock of the goods purchased from outside the State. Therefore, if the dealer wishes to avail the benefit of the scheme, he must ensure that he does not possess stock of such goods as on the date of exercise of option. Advice is to be tendered on above lines.

The selling dealer will not be able to pass on the benefit of the input credit when he opts for the Composition Scheme. A purchasing dealer buying goods from a dealer operating under the Composition Scheme will not get any tax credit for the goods purchased. Hence, as soon as any dealer opts for the Composition Scheme, the VAT chain is broken.
Question 4 (a) The following are the details relating to Mr. Srivatsan, a resident Indian, aged 57, relating to the year ended 31.3.2011 :

76

PAPER 4 : TAXATION

` Income from salaries Loss from house property Loss from cloth business Income from speculation business Loss from specified business covered by section 35AD Long-term capital gains from sale of urban land Long-term capital loss from sale of listed shares in recognized stock exchange (STT paid) Loss from card games Income from betting Life Insurance Premium paid Compute the total income and show the items eligible for carry forward. 2,20,000 1,90,000 2,40,000 30,000 20,000 2,50,000 1,10,000 32,000 45,000 1,20,000 (8 Marks)

(b) State the provisions which enable the Central Government to make rules for administering service tax. For what purposes are such rules made? Name any four such rules issued by the Central Government so far. (4 Marks) (c) What is meant by input tax credit in the context of VAT provisions? How does input tax credit help in achieving the essence of VAT? (4 Marks) Computation of total income of Mr. Srivatsan for the A.Y.2011-12 Particulars Salaries ` `

Answer (a)

Income from salaries


Less:

2,20,000 1,90,000 30,000

Loss from house property

Profits and gains of business or profession

Income from speculation business


Less: Loss from cloth business set off Capital gains

30,000 30,000 Nil

Long-term capital gains from sale of urban land


Less: Loss from cloth business set off

2,50,000 2,10,000 40,000

77

INTEGRATED PROFESSIONAL COMPETENCE EXAMINATION : MAY, 2011

Income from other sources

Income from betting


Gross total income Less: Deduction under section 80C (life insurance premium paid) Total income Losses to be carried forward

45,000
1,15,000

30,000
85,000 `

(1) Loss from cloth business (2,40,000-30,000-2,10,000) (2) Loss from specified business covered by section 35AD
Notes

Nil 20,000

(i)

Long-term capital gains from sale of listed shares in a recognized stock exchange is exempt under section 10(38). Loss from an exempt source cannot be set off against profits from a taxable source. Therefore, long-term capital loss on sale of listed shares cannot be set-off against long-term capital gains from sale of urban land. Loss from specified business covered by section 35AD can be set-off only against profits and gains of any other specified business. Therefore, such loss cannot be set off against any other income. The unabsorbed loss has to be carried forward for set-off against profits and gains of any specified business in the following year.

(ii)

(iii) Business loss cannot be set off against salary income. However, the balance business loss of ` 2,10,000 (` 2,40,000 ` 30,000 set-off against income from speculation business) can be set-off against long-term capital gains of ` 2,50,000 from sale of urban land. Consequently, the taxable long-term capital gains would be ` 40,000. (iv) Loss from card games can neither be set off against any other income, nor can it be carried forward. (v) For providing deduction under Chapter VIA, gross total income has to be reduced by the amount of long-term capital gains and casual income. Therefore, the deduction under section 80C in respect of life insurance premium paid has to be restricted to ` 30,000 [i.e., Gross Total Income of ` 1,15,000 ` 40,000 (LTCG) ` 45,000 (Casual income)]. (vi) Income from betting is chargeable at a flat rate of 30% under section 115BB and no expenditure or allowance can be allowed as deduction from such income, nor can any loss be set-off against such income.
(b) Section 94 of Chapter V and section 96-I of Chapter VA of the Finance Act, 1994 grant powers to Central Government to make rules for carrying out the provisions of these
78

PAPER 4 : TAXATION

Chapters. Rules should be read with the statutory provisions contained in the Act. Rules can never override the Act and cannot be in conflict with the same. So far, the Central Government has issued the following rules for administering service tax (a) Service Tax Rules, 1994 (b) Service Tax (Advance Ruling) Rules, 2003 (c) CENVAT Credit Rules, 2004 (d) Export of Service Rules, 2005 (e) Service Tax (Registration of Special Category of Persons) Rules, 2005 (f) Service Tax (Determination of Value) Rules, 2006 (g) Taxation of Services (Provided from Outside India and Received in India) Rules, 2006 (h) Works Contract (Composition Scheme for Payment of Service Tax) Rules, 2007 (i) (j) Service Tax (Publication of Names) Rules, 2008 Service Tax (Provisional Attachment of Property) Rules, 2008 and

(k) Dispute Resolution Scheme Rules, 2008.


Note: Any four rules may be mentioned. (c) The tax paid by a registered dealer at the earlier point is called input tax. This amount is adjusted/rebated against the tax payable by the purchasing dealer on his sales. This credit availability is called input tax credit (ITC). It can also be referred to as tax credit on a sale within the State or in the course of intra-State trade or commerce.

The essence of VAT is in providing set-off for the tax paid earlier, and this is given effect through the concept of input tax credit/rebate. Thus, input tax credit in relation to any period can be set off by the registered dealer against the amount of his output tax.
Question 5 (a) Mr. Rakesh purchased a house property on 14th April, 1979 for ` 1,05,000. He entered into an agreement with Mr. B for the sale of house on 15th September, 1982 and received an advance of ` 25,000. However, since Mr. B did not remit the balance amount, Mr. Rakesh forfeited the advance. Later on, he gifted the house property to his friend Mr. A on 15th June, 1986. Following renovations were carried out by Mr. Rakesh and Mr. A to the house property: Amount (`) By Mr. Rakesh during FY 1979-80 By Mr. Rakesh during FY 1983-84 By Mr. A during FY 1993-94
79

10,000 50,000 1,90,000

INTEGRATED PROFESSIONAL COMPETENCE EXAMINATION : MAY, 2011

The fair market value of the property as on 1.4.1981 is ` 1,50,000. Mr. A entered into an agreement with Mr. C for sale of the house on 1st June, 1995 and received an advance of ` 80,000. The said amount was forfeited by Mr. A, since Mr. C could not fulfil the terms of the agreement. Finally, the house was sold by Mr. A to Mr. Sanjay on 2nd January, 2011 for a consideration of ` 12,00,000. Compute the capital gains chargeable to tax in the hands of Mr. A for the assessment year 2011-12. Cost inflation indices are as under: Financial Year 1981-82 1983-84 1986-87 1993-94 2010-11 Cost inflation index 100 116 140 244 711 (8 Marks) (b) Briefly explain the provisions relating to advance payment of service tax. (c) What are the major deficiencies of VAT system in India ? Answer (4 Marks) (4 Marks)

(a)

Computation of capital gains chargeable to tax in the hands of Mr. A Particulars Amount (`)

Sale consideration
Less: Indexed cost of acquisition (Note 1) Less: Indexed cost of improvement (Note 2) Long term capital loss Note 1

12,00,000 3,55,500 8,44,500 8,60,114


(15,614)

Indexed cost of acquisition is determined as under: Cost to the previous owner i.e. Mr. Rakesh is ` 1,05,000 Fair Market Value on 1st April, 1981 is ` 1,50,000 Cost to the previous owner or FMV on 1st April, 1981, whichever is more, is to be taken as cost of acquisition of Mr. A
80

1,50,000

PAPER 4 : TAXATION

Less: Advance money forfeited by Mr. A (as per section 51) (Note : Advance forfeited by Mr. Rakesh, the previous owner, should, however, not be deducted)

80,000 70,000
3,55,500

Cost of acquisition
Indexed cost of acquisition (70,000 711/140)

140 is the Cost Inflation Index for F.Y. 1986-87, being the first year in which property is held by Mr. A and 711 is the Cost Inflation Index for F.Y. 2010-11, being the year in which the property is sold.
Note 2

Indexed cost of Improvement is determined as under: Expenditure incurred before 1st April, 1981 should not be considered Expenditure incurred on or after 1st April, 1981 During 1983-84 Indexed cost of Improvement During 1993-94 Indexed cost of Improvement
Total indexed cost of improvement

NIL 3,06,466 5,53,648


8,60,114

[50,000 711/116] [1,90,000 711/244]

(b) Service tax law provides the assessee a facility to make advance payment of service tax on his own volition and adjust the amount so paid against the service tax which he is liable to pay for the subsequent period. Such facility is available when the assessee:

(i)

intimates the details of the amount of service tax paid in advance, to the Jurisdictional Superintendent of Central Excise within a period of 15 days from the date of such payment, and indicates the details of the advance payment made, and its adjustment, if any in the subsequent return to be filed under section 70.

(ii)

(c) The major deficiencies of VAT system in India are as under:

(1) There is lack of uniformity in the rates of VAT in different States. Distortion occurs on account of different rates of VAT, composition scheme, exemptions, difference in classification of goods, etc. (2) Central Sales Tax is not integrated with the State VAT. Therefore, it is difficult to put the purchases from other States at par with the purchases within the State. Consequently, the advantage of neutrality is confined only for purchases within the State.

81

INTEGRATED PROFESSIONAL COMPETENCE EXAMINATION : MAY, 2011

(3) For complying with the VAT provisions, the accounting cost has increased which may not be commensurate with the benefits to traders and small firms. (4) VAT is paid at various stages and not at last stage. This has increased the requirement of working capital and the interest burden on the same. (5) VAT, being a consumption tax, tends to be regressive since the proportion of income spent on consumption is large for the poor than the rich. (6) As a result of introduction of VAT, the administrative cost to the States has increased on account of number of dealers going up significantly.
Note: Any four points may be mentioned. Question 6 (a) Harish Jayaraj Pvt. Ltd. is converted into Harish Jayaraj LLP on 1.1.2011. The following particulars are available to you: ` (i) (ii) (iii) (iv) (v) WDV of land as on 1.4.2010 WDV of machinery as on 1.4.2010 Patents acquired on 1.6.2010 Building acquired on 12.3.2009 for which deduction was allowed under section 35AD. Above building was revalued as on the date of conversion into LLP as 5,00,000 3,30,000 3,00,000 7,00,000 12,00,000

(vi) Unabsorbed business loss as on 1.4.2010 (A.Y. 2007-08) 9,00,000 Though the conversion into LLP took place on 1.1.2011, there was disruption of business and the assets were put into use by the LLP only from 1st March, 2011 onwards. The company earned profits of ` 8 lacs prior to computation of depreciation. Assuming that the necessary conditions laid down in section 47(xiiib) of the Income-tax Act, 1961 have been complied with, explain the tax treatment of the above in the hands of the LLP. (8 Marks) Note WDV of land as on 1.4.2010 may be read as Cost of land. (b) Nigamanth Cargo Handlers Pvt. Ltd. is a cargo handling agency, in existence since 2003. For the quarter ended 31.3.2011, total collections for handling cargo (excluding service tax) was ` 32,00,000. The same included the following receipts also: ` (i) (ii) Handling of cargo containing life saving drugs Handling of export cargo 2,00,000 3,00,000

82

PAPER 4 : TAXATION

(iii) (iv)

Handling of cargo for storage in cold storage

1,00,000

Towards providing service of packing together with transportation of 4,00,000 cargo Ascertain the quantum of taxable cargo handling services for the quarter ended 31.3.2011. Wherever applicable, service tax was charged separately and received in full. (4 Marks) (c) How can a Chartered Accountant help a client in the handling of VAT audit called for by the Department and in conducting external audit of VAT records? (4 Marks) Answer (a) Tax treatment of depreciation and unabsorbed business loss of a private company on its conversion into a LLP

The LLP would be allowed to carry forward and set-off the business loss and unabsorbed depreciation of the predecessor company [Section 72A(6A)].
1. Depreciation

The aggregate depreciation allowable to the predecessor company and successor LLP shall not exceed, in any previous year, the depreciation calculated at the prescribed rates as if the conversion had not taken place. Such depreciation shall be apportioned between the predecessor company and the successor LLP in the ratio of the number of days for which the assets were used by them [Section 32(1)] Therefore, depreciation has to be first calculated as if the conversion had not taken place and then apportioned between the company and the LLP in the ratio of the number of days for which the assets were used by them.
` `

Block I Block II

Machinery Patents

3,30,000 3,00,000

15% 25%

49,500 75,000
1,24,500

Allocation of depreciation

Depreciation on machinery and patents have to be apportioned between the company and the LLP in the ratio of the number of days for which the assets were used by them. Since patents were acquired only on 1.6.2010, it could have been used by the company for 214 days only. Therefore, the depreciation on assets has to be allocated between the company and LLP as follows

83

INTEGRATED PROFESSIONAL COMPETENCE EXAMINATION : MAY, 2011

Company LLP Depreciation Depreciation Total No. of No. of depreciation days of days of for the year usage usage Machinery 49,500 275 44,485 31 5,015 Patents 75,000 214 65,510 31 9,490 1,24,500 1,09,995 14,505 Therefore, depreciation to be allowed in the hands of the company is ` 1,09,995 and depreciation to be allowed in the hands of the LLP is ` 14,505. Asset 2. Unabsorbed business loss to be carried forward by the LLP. Particulars `

Profits of the company before depreciation


Less: Current year depreciation

8,00,000 1,09,995 6,90,005 9,00,000


2,09,995

Business income of the company after depreciation Brought forward business loss Unabsorbed business loss as on 31.12.2010 to be carried forward by the LLP
3. Actual cost of assets to the LLP

(1) The actual cost of the block of assets in case of the LLP shall be the WDV of the block of assets as in the case of the company on the date of conversion. The WDV as on 1.1.2011 for Machinery and Patents are ` 2,85,515 and ` 2,34,490, respectively, which would be the actual cost in the case of the LLP. WDV of Machinery as on 1.1.2011 = 3,30,000 44,485 = ` 2,85,515 WDV of Patents as on 1.1.2011 = 3,00,000 65,510 = ` 2,34,490 (2) Land is not a depreciable asset. The cost of acquisition of land to the LLP would be the cost for which the company acquired it, as increased by the cost of improvement. (3) In respect of the building, deduction had been allowed in the earlier year under section 35AD. Hence, there is no question of depreciation during the current year. The actual cost of the building to the LLP would be Nil.
(b) Quantum of taxable cargo handling services for the quarter ended 31.3.2011:Particulars Amount (`) Amount (`)

Total collections for handling cargo (excluding service tax)


84

32,00,000

PAPER 4 : TAXATION

Less:

(i) (ii) (iii) (iv)

Receipts for handling of cargo containing life saving drugs [Refer Note 1] Receipts for handling of export cargo [Refer Note 2] Receipts for handling of cargo for being stored in cold storage [Refer Note 3] Receipts towards providing service of packing together with transportation of cargo [Refer Note 4] 4,00,000
Value of taxable services 28,00,000

3,00,000 1,00,000

Notes:

1. 2. 3.

Handling of cargo containing life saving drugs is taxable service as there is no specific exemption available for such activity. The definition of cargo handling service specifically excludes handling of export cargo. Cargo handling service provided in relation to goods intended to be stored in a cold storage is exempt from payment of service tax vide Notification No. 10/2002 ST dated 01.08.2002. Service of packing together with transportation of cargo is covered by the definition of cargo handling service. Hence, the same is liable to service tax and no adjustment is needed.

4.

(c) Handling audit by Departmental Auditors

There are audit wings in VAT Departments and certain percentage of dealers are taken up for audit every year on scientific basis. Chartered Accountants can ensure proper record keeping to satisfy the Departmental auditors. The professional expertise of a Chartered Accountant will help him in effectively replying audit queries and sorting out audit objections.
External audit of VAT records

Under VAT system, trust has been reposed on tax payers as there will be no regular assessment of all VAT returns, but only few returns will be scrutinized. In other cases, returns filed by dealers will be accepted. Thus, a check on compliance becomes necessary. Chartered Accountants can play a very vital role in ensuring tax compliance by audit of VAT accounts.
Question 7 (a) Answer any two of the following three sub divisions:
85

INTEGRATED PROFESSIONAL COMPETENCE EXAMINATION : MAY, 2011

(1) Specify the persons who are authorized to sign and verify under section 140, the return of income filed under section 139 of the Income-tax Act, 1961 in the case of: (i) (ii) (iii) (iv) Political party; Local authority; Association of persons, and Limited Liability Partnership (LLP). (4 Marks)

(2) The following details have been furnished by Mrs. Hemali pertaining to the year ended 31.3.2011 : (i) Cash gift of ` 51,000 received from her friend on the occasion of her Shastiaptha Poorthi, a wedding function celebrated on her husband completing 60 years of age. This was also her 25th wedding anniversary. On the above occasion, a diamond necklace worth ` 2 lacs was presented by her sister living in Dubai.

(ii)

(iii) When she celebrated her daughter's wedding on 21.2.2011, her friend assigned in Mrs. Hemali's favour, a fixed deposit held by the said friend in a scheduled bank; the value of the fixed deposit and the accrued interest on the said date was ` 51,000. Compute the income, if any, assessable as income from other sources. (4 Marks) (3) During the financial year 2010-11, the following payments/expenditure were made/incurred by Mr. Yuvan Raja, a resident individual (whose turnover during the year ended 31.3.2010 was ` 39 lacs) : (i) (ii) Interest of ` 12,000 was paid to Rehman & Co., a resident partnership firm, without deduction of tax at source; Interest of ` 4,000 was paid as interest to Mr. R.D. Burman, a non-resident, without deduction of tax at source;

(iii) ` 3,00,000 was paid as salary to a resident individual without deduction of tax at source; (iv) He had sold goods worth ` 5 lacs to Mr. Deva. He gave Mr. Deva a cash discount of ` 12,000 later. Commission of ` 15,000 was paid to Mr. Vidyasagar on 2.7.2010. In none of these transactions, tax was deducted at source. Briefly discuss whether any disallowance arises under the provisions of section 40(a)(i)/40(a)(ia) of the Income-tax Act, 1961. (4 Marks) (b) State the due dates for filing of service tax returns. Will the delayed filing of service tax return result in payment of any late fee? If so, how much? (4 Marks) (c) Briefly list out the contents of VAT invoice.
86

(4 Marks)

PAPER 4 : TAXATION

Answer (a) (1) The following persons (mentioned in Column III below) are authorised as per section 140, to sign and verify the return of income filed under section 139. I II III

(i) (ii) (iii) (iv)

Political party Local authority Association Persons LLP of

Chief Executive Officer of such party (whether known as secretary or by any other designation). Principal Officer thereof. Any member of the association or the principal officer thereof. Designated partner, or Any partner, where the designated partner is not able to sign and verify the return for any unavoidable reason; where there is no designated partner.

(2) (i)

Any sum of money received by an individual on the occasion of the marriage of the individual is exempt. This provision is, however, not applicable to a cash gift received during a wedding function celebrated on completion of 60 years of age. The gift of ` 51,000 received from a non-relative is, therefore, chargeable to tax under section 56(2)(vii) in the hands of Mrs. Hemali.

(ii)

The provisions of section 56(2)(vii) are not attracted in respect of any sum of money or property received from a relative. Thus, the gift of diamond necklace received from her sister is not taxable under section 56(2)(vii), even though jewellery falls within the definition of property.

(iii) To be exempt from applicability of section 56(2)(vii), the property should be received on the occasion of the marriage of the individual, not that of the individuals son or daughter. Therefore, this exemption provision is not attracted in this case. Any sum of money received without consideration by an individual is chargeable to tax under section 56(2)(vii), if the aggregate value exceeds ` 50,000 in a year. Sum of money has, however, not been defined under section 56(2)(vii). Therefore, there are two possible views in respect of the value of fixed deposit assigned in favour of Mrs. Hemali (1) The first view is that fixed deposit does not fall within the meaning of sum of money and therefore, the provisions of section 56(2)(vii) are not

87

INTEGRATED PROFESSIONAL COMPETENCE EXAMINATION : MAY, 2011

attracted. It may be noted that fixed deposit is also not included in the definition of property. (2) However, another possible view is that fixed deposit assigned in favour of Mrs. Hemali falls within the meaning of sum of money received.
Income assessable as Income from other sources

If the first view is taken, the total amount chargeable to tax as Income from other sources would be ` 51,000, being cash gift received from a friend on her Shastiaptha Poorthi. As per the second view, the provisions of section 56(2)(vii) would be attracted in respect of the fixed deposit assigned and the Income from other sources of Mrs. Hemali would be ` 1,02,000 (` 51,000 + ` 51,000).
(3) Disallowance under section 40(a)(i)/40(a)(ia) of the Income-tax Act, 1961 is attracted where the assessee fails to deduct tax at source as is required under the Act, or having deducted tax at source, fails to remit the same to the credit of the Central Government within the stipulated time limit.

The assessee is a resident individual, who was not subjected to tax audit during the immediately preceding previous year i.e., P.Y.2009-10 (as his turnover is less than ` 40 lakh in that year) and the TDS obligations have to be considered bearing this in mind. (i) The obligation to deduct tax source from interest paid to a resident arises under section 194A in the case of an individual, only where he was subject to tax audit under section 44AB in the immediately preceding previous year, i.e., P.Y.2009-10. From the data given, it is clear that he was not subject to tax audit under section 44AB in the P.Y.2009-10. Hence, disallowance under section 40(a)(ia) is not attracted in this case. In the case of interest paid to a non-resident, there is obligation to deduct tax at source under section 195, hence non-deduction of tax at source will attract disallowance under section 40(a)(i).

(ii)

(iii) Disallowance under section 40(a)(ia) is not attracted for failure to deduct tax at source under section 192 from salaries. (iv) The obligation to deduct tax at source under section 194-H from commission paid in excess of ` 5,000 w.e.f. 1.7.2010 to a resident arises in the case of an individual, only where he was subject to tax audit under section 44AB in the immediately preceding previous year. From the data given, it is clear that he was not subject to tax audit under section 44AB in the P.Y.2009-10. Hence, there is no obligation to deduct tax at source under section 194H during the P.Y. 2010-11. Therefore, disallowance under section 40(a)(ia) is not attracted in this case.
88

PAPER 4 : TAXATION

(b) The service tax return (in Form ST-3) should be filed on half yearly basis by the 25th of the month following the particular half-year. The due dates on this basis are as under: Half year Due date

1st April to 30th September 1st October to 31st March

25th October 25th April

In case the due date of filing of return falls on a public holiday, the assessee can file the return on the immediately succeeding working day. Yes, late fee will be levied for delay in furnishing of the service tax return. prescribed late fee is given hereunder:
S. No. Period of delay Particulars 15 days from the date prescribed for submission of the return Beyond 15 days but not later than 30 days from the date prescribed for submission of the return. Beyond 30 days from the date prescribed for submission of the return Late fee ` 500

The

(a) (b) (c)

1,000

An amount of ` 1,000 plus ` 100 for every day from the 31st day till the date of furnishing the said return. However, the total late fee for delayed submission should not exceed ` 2,000.
(c) VAT legislations of all the States provide for the tax invoice. Generally, the various legislations provide that the tax invoice should have the following contents:

(i) (ii) (iii) (iv) (v) (vi) (vii) (viii) (ix) (x)

the words tax invoice in a prominent place; name and address of the selling dealer; registration number of the selling dealer; name and address of the purchasing dealer; registration number of the purchasing dealer (may not be required under all VAT legislations); pre-printed or self-generated serial numbers; date of issue; description, quantity and value of goods sold; rate and amount of tax charged in respect of taxable goods; signature of the selling dealer or his regular employee duly authorized by him for such purposes.

89

SUMMARY OF EXAMINERS COMMENTS ON THE PERFORMANCE OF THE CANDIDATES PAPER 1 : ACCOUNTING General Comments As contemplated by the examiners, spelling mistakes and poor usage of grammar was the common problem among the candidates. Also, most of the answers lacked proper presentation, good expression, clarity and precision. It is observed that the candidates generally neglect basic accounting concepts. They should lay stress on theoretical aspects of each and every topic. Many candidates failed to support their calculations with proper working notes. Candidates are suggested to strictly follow the instructions given in the question paper. Short length questions with reasoning, intricately test the candidates understanding of the topics covered under the study material. Therefore, candidates are advised to work hard on such questions. It is also suggested that the candidates should go through the study material thoroughly. Specific Comments Question 1. In part (a) of the question, many candidates added liquidation expenses for the purpose of calculating purchase consideration. In part (b), most of the candidates showed lack of conceptual understanding relating to adjustment for Goods sold on approval basis for the purpose of calculation of loss of stock and its insurance claim. Some candidates treated Goods sold on approval basis as sales and some added it with closing stock destroyed by fire. In part (d), few candidates were not able to present the investment account in a proper manner. Question 2. Share of goodwill distributed among new and old partners was wrongly calculated by few candidates. Revaluation account was inversely prepared by several candidates. Many candidates directly reduced 1/3rd from old share of Amit and 1/4th from old share of Sumit to arrive at new profit sharing ratio instead of reducing their 1/3rd and 1/4th share respectively. Question 3. Most of the candidates erred in the calculation of purchase consideration. Majority of the candidates added the reimbursement of liquidation expenses to purchase consideration. Question 4. Not many candidates were able to correctly compute the amount of income tax paid during the year. Some candidates considered ` 2,50,000 as cash-in-flow from equity shares instead of ` 1,50,000. Question 6. Many candidates prepared cash and bank account jointly, thereby making mistake in calculation of cash defalcated by the cashier. Some failed to understand that the cash defalcated by the cashier is a loss to the entity and therefore erred in the calculation of correct net profit. Question 7. In part (a) of the question, few candidates erred in the calculation of number of days from the base date. Some candidates took 30th June as base date but while calculating

SUMMARY OF EXAMINERS COMMENTS

the average due date, they have adopted the formula of average due date on the basis of base date as the starting date. In part (c), most of the candidates could not give the necessary journal entries for adjustment of goodwill on retirement and admission of a partner. In part (e), items not to be posted in the Debtors Total Account were not supported with proper reasoning. Some candidates wrongly entered bad debts recovered and Bills receivable endorsed into Total Debtors Account. PAPER 2 : BUSINESS LAWS, ETHICS AND COMMUNICATION General Comments 1. 2. The performance in general was satisfactory. Considering the level of knowledge that is expected from the candidates in this paper i.e. working knowledge, candidates are required to write answers based on provisions of the law, wherever necessary and in this connection, they are advised to read the study material. Candidates are further advised to read the subject with a practical orientation and writing answers based on general view should be avoided.

3.

Specific Comments Question 1.(a) Instead of writing answer on the determination of compensation on the breach of contract, candidates described different types of breach of contract and provisions as contained in Section 73 of the Indian Contract Act, 1872 were not known to many. (b) & (c) The performance to the multiple objective questions was found excellent. (d) On the law and procedure relating to registration of a non-profit organisation as a company under the Companies Act, 1956, answers were general in nature and provisions of section 25 were not mentioned. Question 2.(a) The problem based on provisions as contained in the Payment of Bonus Act, 1965 was well answered by many candidates, though they could refer the relevant case law and section in the said Act. (b) In stating the elements which create discrimination in employment in the business organisations, answers were in general and the performance was average. Question 3.(a) While the candidates correctly mentioned the amount of gratuity payable to employees in a seasonal as well as other establishments, under the payment of Gratuity Act, 1972, the basis on which it is calculated were not correctly written. Amendment in the ceiling regarding maximum gratuity payable was correctly mentioned by many candidates. (b) Tips for improving inter-personal skills in a business organisation were not specific and it is evident that candidates did not read the study material. Question 4. The performance to the question in general was average. Candidates did not explain the purposes for which the Securities Premium Account can be used. On the second
91

INTEGRATED PROFESSIONAL COMPETENCE EXAMINATION : MAY, 2011

part of the question relating to ethics, it seems that candidates are having some difficulty in answers, either due to less attention being paid or in understanding the subject. Question 5. The performance of the candidates in general to the various sub-parts of the question was satisfactory. Question 6. The procedure for converting a private company into a public company under the provisions of the Companies Act, 1956 were not listed out in order and they were general in nature. In respect of questions relating to guidelines for managing ethics, need for whistleblowing in the work place and characteristics of group personality, very few candidates could write correct answers and many of them even did not attempted it. Question 7. On the composition and functions of the Central Board of Trustees under E.P.F & M.P. Act, 1952, very few could write correct answers. On the documents to be filed with ROC at the time of registration of a company under the Companies Act, 1956, many of them were not aware of documents other than Memorandum and Articles of Association. Based on performance to this question and other related questions related to company law portion, candidates have to equip their working knowledge in the subject. The performance to the second alternative choice question on company law and ethics was average. The answers to the third alternative choice questions based on ethics was not upto the mark. The fourth alternative choice questions relating to communication were written in general. PAPER 3 : COST ACCOUNTING AND FINANCIAL MANAGEMENT General Comments Overall performance of the candidates was average. Performance revealed a lack of knowledge of concepts in the areas of Cost Accounting and Financial Management as expected of a professional student of Accountancy course. The candidates need to understand the subjects conceptually and also fine tune their presentation skills. Further, some candidates attempted different parts of the same question at random instead of answering them consecutively. Specific Comments Question 1.(a) This question related to computation of operating and financial leverage for two plans. Majority of the candidates performed well except a few of them, who failed to show the calculations separately for the two plans under the given situations. (b) Problem based on calculation of remuneration of the workers under Halsey and Rowen plans. Since many candidates could not arrive at the time saved correctly, therefore, were not able to calculate the correct remuneration. (c) This question requires preparation of stores ledger. Since it is also a compulsory question almost all the candidates tried to prepare the ledger accounts but many candidates
92

SUMMARY OF EXAMINERS COMMENTS

could not prepare it since they were confused with the methods to be adopted though the date is clearly mentioned in the problem. Most of the candidates ignored the format while preparing the store ledger. (d) Short note on factoring and its advantages was discussed by many candidates on correct lines. However, a small percentage of the candidates explained the concept of present value tables or factors of working capital management instead. Question 2.(a) Problem on machine hour rate was not attempted well by many candidates since they could not calculate the effective working hours. Apportionment was made by almost all the candidates. (b) The question required the candidates to advise the management regarding adoption of new credit policy. Majority of the candidates have well attempted the question and have advised correctly. However, few of them have wrongly calculated investment in receivables on the basis of total sales instead of total cost. Question 3. A large percentage of the candidates have attempted this problem on Preparation of estimated working capital statement but only a few could attempt it on correct lines. Only a few of the candidates have calculated the cash cost of sales correctly and majority have computed debtors on the basis of sales. Question 4. Majority of the candidates prepared the Cash Flow Statement on correct lines but failed to present it in the proper format. However, a few candidates could not work out cash flow from operating activities correctly and only a few have computed the cash and cash equivalent at the beginning as well as at the end of the year. Further, the candidates have not presented their answers supported by proper working notes in preparation of cash flow statement. Question 5.(a) This problem was on cost ledger. No candidate was able to calculate the profit under both costing and financial methods. Almost all the candidates have just opened the head of relevant accounts but the figures given are irrelevant. (b) Majority of the candidates were not able to discuss debt securitisation well and gave answers based on security for a debt or loan. Question 6. (a) This question on Computation of earnings per share was well attempted by many candidates. Majority of the candidates computed indifference points between different proposals on correct lines. However, few of the candidates faltered in computation of total shares. (b) This theory question on cost unit and cost centers are generally discussed without correct definition and proper points. Question 7.(a) Packing expenses and Fringe benefits were generally explained by the candidates but without indicating the requirement of treatment. (b) A large percentage of the candidates explained Operating Profit ratio and Operating leverage instead of Operating ratio. (c) Many candidates explained the meaning without giving the causes of labour turnover.
93

INTEGRATED PROFESSIONAL COMPETENCE EXAMINATION : MAY, 2011

(d) Majority of the candidates explained correctly William J. Baumal versus Miller Orr cash management model. (e) Process of estimating profit/loss on incomplete contracts was given correctly by many candidates. PAPER 4 : TAXATION Specific Comments Question 1.(a) The question is based on computation of capital gain in case of transfer of shares by an individual to his spouse without consideration, income earned thereon and subsequent sale of shares at concessional price by the spouse to her friend. A number of adjustments were asked on computation of long term capital gain, short term capital gain, clubbing of income and provisions of section 56(2)(vii). Students were found to be lacking on provisions of section 56 as well as on applicability of clubbing provisions in relation to accretion to the assets transferred to spouse. Some students have wrongly treated profit on sale of bonus shares as long term capital gains. Students have not written about the nontaxability of the difference of ` 40,000 (` 2,10,000 ` 1,70,000) in the fair market value and actual sale consideration of shares in the hands of Mrs. Hema on account of the difference not exceeding ` 50,000. (b) Only few have answered this question correctly. Majority of the students have wrongly taken the total profits derived from the Unit S i.e. ` 13 lacs as fully exempt on account of the unit being in SEZ, instead calculating the profits in relation to export turnover of Unit S. (c) In many cases, students calculated value of taxable service but omitted to calculate service tax payable. They also ignored the fact that since the firm had rendered taxable service of ` 20.2 lakh in the previous financial year, it is not a small service provider. Further, many students have wrongly considered the amount collected from companies for prerecruitment screening as well as for recruitment of permanent and temporary staff as inclusive of service tax. Also, many of them have wrongly taken service tax liability as nil in case of advance received. (d) Only a very few students were able to compute the net VAT liability correctly. While some of them did not consider the VAT credit on closing stock of raw materials, others allowed the whole credit on capital goods in one year. Some students also did not consider the credit on capital goods at all. Further, students also failed to give reasons for excluding other items provided in the question which were not relevant for computation of input credit. Question 2.(a)(i) The question is on taxability of payment made outside India to a nonresident for the consultancy services related to a project in India. The question is to be answered based on the amendment made by the Finance Act, 2010 in section 9 of the Income-tax Act, 1961. Since the question did not specify chargeability under the Income-tax Act, 1961, some students have answered the same on the basis of service tax provisions related to import of services
94

SUMMARY OF EXAMINERS COMMENTS

(ii) The most common mistake committed by students was that they had applied weighted deduction @200% under section 35(2AB) (applicable to companies) for in-house research and development expenditure instead of 100% under section 35(1) (applicable for individual). (b) Though the question clearly provided that service tax is collected separately on coaching fees of ` 14,50,000 collected from students, many students wrongly assumed it to be inclusive of service tax and made the calculations accordingly. Also, few of them wrongly considered the value of free coaching to be liable to service tax. A large number of students failed to charge service tax on the advance received during the quarter ended 30.06.2010 which was returned subsequently. (c) While the first part of the question was answered correctly by a large number of students, only a handful of them were able to answer the second part relating to services being included in the VAT net. Question 3.(a) The common mistakes committed by students while answering this question are:(i) Most of the students have wrongly taken the entire interest of ` 1,50,000 on capital received from Oscar Musicals & Co. whereas the same must be restricted to ` 1,20,000 which is the maximum interest allowable in the hands of the firm. Many students have considered net interest of ` 13,500 on fixed deposit as income instead of gross interest of ` 15,000.

(ii)

(iii) Several students have taken the entire income tax refund of ` 34,500 received for AY 2009-10 as income under the head Income from Other Sources instead of considering only the interest of ` 2,300. (iv) Many students have not applied the correct rate of depreciation in case of Computer. (v) Only few students have correctly worked out the WDV of Motor Car. (b)(i) It has been observed that majority of the students have simply answered as I agree without giving suitable explanations for such agreement. Question 4.(a) (i) (ii) Most of the students were not clear regarding the provisions of set off of losses. Many students have wrongly set off loss of ` 32,000 from card games against income of ` 45,000 from betting.

(iii) Some students have wrongly set off long term capital loss from sale of listed shares in a recognized stock exchange against long term capital gain from sale of urban land. They were not aware that loss from an exempt source cannot be set off against gains from a taxable source. (iv) Few students have wrongly exempted long term capital gain from sale of urban land. (v) Some of the students have wrongly set off loss from specified business covered under section 35AD against income from speculation business.
95

INTEGRATED PROFESSIONAL COMPETENCE EXAMINATION : MAY, 2011

(b) This question was based on the general understanding to administer service tax laws and to name such rules issued by the Central Government. Students did not write the name of the rules correctly and secured partial marks for general understanding. (c) Many students answered the first part correctly. However, they failed to explain as to how input tax credit helps in achieving the essence of VAT and instead wrote long answers explaining the advantages of input tax credit. Question 5.(a) 1. 2. Some students have wrongly considered ` 1,05,000 as the cost of the house property, instead of adopting the fair market value of ` 1,50,000 on 01.04.1981, which is higher. Many students have wrongly deducted the amount of ` 25,000 forfeited by Mr. Rakesh to arrive at the cost to be considered for indexation.

(b) Some students wrongly answered this question on the basis of income-tax provisions. Few of them also wrongly mentioned the time limit of intimating the details of service tax paid in advance to jurisdictional Superintendent of Central Excise as 30 days from the date of adjustment of the service tax in the subsequent period instead of 15 days from the date of advance payment. Question 6.(a) Most students have failed to calculate the allocation of depreciation between private limited company and LLP correctly. In case of depreciation on Patents and Machinery the number of days of usage for the LLP has been calculated as 3 months instead of 31 days and for the private limited company as 275 days instead of 214 days (for depreciation on patents). Almost all the candidates have provided depreciation for building without considering that deduction under section 35AD has already been allowed in an earlier year. (b) Students were required to calculate the value of taxable cargo handling services in this question. Many students simply ignored item no. (i) & (iv) without mentioning that these are covered under cargo handling services and hence no adjustment is needed from total collections of ` 32,00 lakh. (c)- Students wrote long essay types answer for this question instead of mentioning the specific areas in relation to VAT audit where a Chartered Accountant could play a vital role. Question 7.(a)(1) Majority of candidates have answered this question incorrectly. A lot of guess work has gone in answering this question. (a)(3) This question is to test the applicability of TDS provisions and allowability of payments/ expenditures made by a resident individual to residents and non-residents. The question was answered by a very few candidates due to Iack of knowledge and their answers were very general. Candidates were not aware of the fact that since the assessee was not subject to tax audit in the immediately preceding financial year hence, the question of disallowance of interest and commission payment in item nos (i) & (iv), respectively, does not arise. They were also not aware that salary is not an item figuring in Section 40(a)(ia) for disallowance. (b) Many students wrongly mentioned the due dates for payment of service tax instead of writing the due dates for filing of service tax returns which was required by the question.
96

You might also like